Exam 3

¡Supera tus tareas y exámenes ahora con Quizwiz!

A client with diabetes mellitus who takes insulin is seen in the health care clinic. The client tells the clinic nurse that after the insulin injection, the insulin seems to leak through the skin. The nurse would appropriately determine the problem by asking the client which question? 1. "Are you rotating the injection site?" 2. "Are you aspirating before you inject the insulin?" 3. "Are you using a 1-inch needle to give the injection?" 4. "Are you placing an air bubble in the syringe before injection?"

1. "Are you rotating the injection site?"

The home health nurse visits a client with a diagnosis of type 1 diabetes mellitus. The client relates a history of vomiting and diarrhea and tells the nurse that no food has been consumed for the last 24 hours. Which additional statement by the client indicates a need for further teaching? 1. "I need to stop my insulin." 2. "I need to increase my fluid intake." 3. "I need to monitor my blood glucose every 3 to 4 hours." 4. "I need to call the health care provider (HCP) because of these symptoms."

1. "I need to stop my insulin."

The nurse is taking a health history for a client with hyperparathyroidism. Which question would elicit information about this client's condition? 1. "Do you have tremors in your hands?" 2. "Are you experiencing pain in your joints?" 3. "Do you notice swelling in your legs at night?" 4. "Have you had problems with diarrhea lately?"

2. "Are you experiencing pain in your joints?"

A nurse is instructing a client with DM II regarding the patho of the client's condition. What statement made by the client shows that learning is taking place?

my cells are resistant to the effects of insulin."

A nurse is caring for a client with DM II. What findings indicate to the nurse of hyperglycemia?

sweating and increased urination

A client received 5 units of aspart insulin (NovoLog) subcutaneously just before eating lunch at 12:00 pm. The nurse should assess the client for a hypoglycemic reaction at which times? 1. Between 1:00 and 3:00 pm 2. 10 minutes after administration 3. Between 4:00 pm and 12:00 am 4. Between 8:00 pm and 10:00 pm

1. Between 1:00 and 3:00 pm

The nurse is providing dietary instructions to help with diabetes control for a client newly diagnosed with diabetes mellitus who will be taking insulin. The nurse should provide the client with which best instruction? 1. Eat meals at approximately the same time each day. 2. Adjust meal times depending on blood glucose levels. 3. Vary meal times if insulin is not administered at the same time every day. 4. Avoid being concerned about the time of meals so long as snacks are taken on time.

1. Eat meals at approximately the same time each day.

A client arrives in the hospital emergency department in an unconscious state. As reported by the spouse, the client has diabetes mellitus and began to show symptoms of hypoglycemia. A blood glucose level is obtained for the client, and the result is 40 mg/dL. Which medication should the nurse anticipate to be prescribed for the client? 1. Glucagon 2. Humulin N insulin 3. Humulin R insulin 4. Glyburide (DiaBeta)

1. Glucagon

A client with diabetes mellitus who refuses to take insulin as prescribed exhibits markedly increased blood glucose levels after a meal. The nurse caring for the client anticipates that which initial body response to elevated glucose levels will worsen the situation for the client? 1. Glycogenolysis 2. Gluconeogenesis 3. Binding of glucose onto cell membranes 4. Transport of glucose across cell membranes

1. Glycogenolysis

The nurse is caring for a postoperative parathyroidectomy client. Which client complaint would indicate that a life-threatening complication may be developing, requiring notification of the health care provider immediately? 1. Laryngeal stridor 2. Abdominal cramps 3. Difficulty in voiding 4. Mild to moderate incisional pain

1. Laryngeal stridor/laryngeal nerve damage

A nurse is reviewing the health care provider's prescriptions for a client diagnosed with hypothyroidism. Which medication prescription should the nurse question and verify? 1. Morphine sulfate 2. Docusate sodium (Colace) 3. Acetaminophen (Tylenol) 4. Levothyroxine sodium (Synthroid)

1. Morphine sulfate

The nurse is monitoring a client newly diagnosed with diabetes mellitus for signs of complications. Which sign, if exhibited in the client, would indicate hyperglycemia? 1. Polyuria 2. Diaphoresis 3. Hypertension 4. Increased pulse rate

1. Polyuria

The nurse should include which interventions in the plan of care for a client with hypothyroidism? Select all that apply. 1. Provide a warm environment for the client. 2. Instruct the client to consume a low-fat diet. 3. A thyroid-releasing inhibitor will be prescribed. 4. Encourage the client to consume a well-balanced diet. 5. Instruct the client that thyroid replacement therapy will be needed. 6. Instruct the client that episodes of chest pain are expected to occur.

1. Provide a warm environment for the client. 2. Instruct the client to consume a low-fat diet. 4. Encourage the client to consume a well-balanced diet. 5. Instruct the client that thyroid replacement therapy will be needed.

A client's serum blood glucose level is 48 mg/dL. The nurse would expect to note which as an additional finding when assessing this client? 1. Slurred speech 2. Increased thirst 3. Increased appetite 4. Increased urination

1. Slurred speech

A client with diabetes mellitus is being discharged following treatment for hyperglycemic hyperosmolar state (HHS) precipitated by acute illness. The client tells the nurse, "will call the health care provider (HCP) the next time I can't eat for more than a day or so." Which statement reflects the most appropriate analysis of this client's level of knowledge? 1. The client needs immediate education before discharge. 2. The client requires follow-up teaching regarding the administration of oral antidiabetics. 3. The client's statement is inaccurate, and he or she should be scheduled for outpatient diabetic counseling. 4. The client's statement is inaccurate, and he or she should be scheduled for educational home health visits.

1. The client needs immediate education before discharge.

A client with an endocrine disorder complains of weight loss and diarrhea, and says that he can "feel his heart beating in his chest." The nurse interprets that which gland is most likely responsible for these symptoms? 1. Thyroid 2. Pituitary 3. Parathyroid 4. Adrenal cortex

1. Thyroid

The nurse is monitoring a client who was diagnosed with type 1 diabetes mellitus and is being treated with NPH and regular insulin. Which client complaints would alert the nurse to the presence of a possible hypoglycemic reaction? Select all that apply. 1. Tremors 2. Anorexia 3. Irritability 4. Nervousness 5. Hot, dry skin 6. Muscle cramps

1. Tremors 3. Irritability 4. Nervousness

The nurse has provided instructions to the client with hyperparathyroidism regarding home care measures to manage the symptoms of the disease. Which statement by the client indicates a need for further instruction? 1. "I should avoid bed rest." 2. "I need to avoid doing any exercise at all." 3. "I need to space activity throughout the day." 4. "I should gauge my activity level by my energy level."

2. "I need to avoid doing any exercise at all."

The nurse has provided home care measures to the client with diabetes mellitus regarding exercise and insulin administration. Which statement by the client indicates a need for further instruction? 1. "I should always wear a Medic-Alert bracelet." 2. "I should perform my exercise at peak insulin time." 3. "I should always carry a quick-acting carbohydrate when I exercise." 4. "I should avoid exercising at times when a hypoglycemic reaction is likely to occur."

2. "I should perform my exercise at peak insulin time."

The family of a bedridden client with type 2 diabetes mellitus and a chronic kidney disease calls a nurse to report symptoms of headache, polydipsia, and increased lethargy. Which most important question should the nurse ask the family to determine a possible problem? 1. "What is the client's urine output?" 2. "What is the client's capillary blood glucose level?" 3. "Has there been any change in the dietary intake?" 4. "Have you increased the amount of fluids provided?"

2. "What is the client's capillary blood glucose level?"

The home care nurse is visiting a client newly diagnosed with diabetes mellitus. The client tells the nurse that he is planning to eat a dinner meal at a local restaurant this week. He asks the nurse if eating at a restaurant will affect diabetic control and if this is allowed. Which nursing response is most appropriate? 1. "You are not allowed to eat in restaurants." 2. "You should order a half-portion meal and have fresh fruit for dessert." 3. "If you plan to eat in a restaurant, you need to skip the lunchtime meal." 4. "You should increase your daily dose of insulin by half on the day that you plan to eat in the restaurant."

2. "You should order a half-portion meal and have fresh fruit for dessert."

The nursing instructor asks a nursing student to identify the risk factors associated with the development of thyrotoxicosis. The student demonstrates understanding of the risk factors by identifying an increased risk for thyrotoxicosis in which client? 1. A client with hypothyroidism 2. A client with Graves' disease who is having surgery 3. A client with diabetes mellitus scheduled for a diagnostic test 4. A client with diabetes mellitus scheduled for débridement of a foot ulcer

2. A client with Graves' disease who is having surgery

The nurse is performing an assessment on a client with a diagnosis of hyperthyroidism. Which assessment finding should the nurse expect to note in this client? 1. Dry skin 2. Bulging eyeballs 3. Periorbital edema 4. Coarse facial features

2. Bulging eyeballs

A client with diabetes mellitus demonstrates acute anxiety when first admitted to the hospital for the treatment of hyperglycemia. What is the most appropriate intervention to decrease the client's anxiety? 1. Administer a sedative. 2. Convey empathy, trust, and respect toward the client. 3. Ignore the signs and symptoms of anxiety so that they will soon disappear. 4. Make sure that the client knows all the correct medical terms to understand what is happening.

2. Convey empathy, trust, and respect toward the client.

The nurse is preparing a plan of care for a client with diabetes mellitus who has hyperglycemia. The nurse places highest priority on which client problem? 1. Lack of knowledge 2. Inadequate fluid volume 3. Compromised family coping 4. Inadequate consumption of nutrients

2. Inadequate fluid volume

The nurse is providing instructions to a client newly diagnosed with diabetes mellitus. The nurse gives the client a list of the signs of hyperglycemia. Which specific sign of this complication should be included on the list? 1. Shakiness 2. Increased thirst 3. Profuse sweating 4. Decreased urine output

2. Increased thirst

A client with diabetes mellitus is at risk for a serious metabolic disorder from the breakdown of fats for conversion to glucose. The nurse plans care for the client, knowing that pathological fat metabolism is occurring if the client has elevated levels of which substance? 1. Glucose 2. Ketones 3. Glucagon 4. Lactate dehydrogenase

2. Ketones

A client is admitted to an emergency department, and a diagnosis of myxedema coma is made. Which action would the nurse prepare to carry out initially? 1. Warm the client. 2. Maintain a patent airway. 3. Administer thyroid hormone. 4. Administer fluid replacement.

2. Maintain a patent airway.

The nurse has developed a postoperative plan of care for a client who had a thyroidectomy and documents that the client is at risk for developing an ineffective breathing pattern. Which nursing intervention should the nurse include in the plan of care? 1. Maintain a supine position. 2. Monitor neck circumference every 4 hours. 3. Maintain a pressure dressing on the operative site. 4. Encourage deep breathing exercises and vigorous coughing exercises.

2. Monitor neck circumference every 4 hours.

A client has overactivity of the thyroid gland. The nurse plans care, knowing that the client will experience which effects from this hormonal excess? 1. Weight gain 2. Nutritional deficiencies 3. Low blood glucose levels 4. Increased body fat stores

2. Nutritional deficiencies

The nurse is caring for a client who is 2 days postoperative following an abdominal hysterectomy. The client has a history of diabetes mellitus and has been receiving regular insulin according to capillary blood glucose testing four times a day. A carbohydrate-controlled diet has been prescribed but the client has been complaining of nausea and is not eating. On entering the client's room, the nurse finds the client to be confused and diaphoretic. Which action is most appropriate at this time? 1. Call a code to obtain needed assistance immediately. 2. Obtain a capillary blood glucose level and perform a focused assessment. 3. Ask the unlicensed assistive personnel (UAP) to stay with the client while obtaining 15 to 30 g of a carbohydrate snack for the client to eat. 4. Stay with the client and ask the UAP to call the health care provider (HCP) for a prescription for intravenous 50% dextrose.

2. Obtain a capillary blood glucose level and perform a focused assessment.

The nurse is completing an assessment on a client who is being admitted for a diagnostic workup for primary hyperparathyroidism. Which client complaint would be characteristic of this disorder? 1. Diarrhea 2. Polyuria 3. Polyphagia 4. Weight gain

2. Polyuria

The nurse caring for a client with a diagnosis of hypoparathyroidism reviews the laboratory results of blood tests for this client and notes that the calcium level is extremely low. The nurse should expect to note which on assessment of the client? 1. Unresponsive pupils 2. Positive Trousseau's sign 3. Negative Chvostek's sign 4. Hyperactive bowel sounds

2. Positive Trousseau's sign

The nurse is caring for a client admitted to the hospital with uncontrolled type 1 diabetes mellitus. In the event that diabetic ketoacidosis (DKA) does occur, the nurse anticipates that which medication would most likely be prescribed? 1. Glucagon 2. Regular insulin 3. Glyburide (DiaBeta) 4. Neutral protamine Hagedorn (NPH) insulin

2. Regular insulin

The nurse teaches a client with diabetes mellitus about differentiating between hypoglycemia and ketoacidosis. The client demonstrates an understanding of the teaching by stating that a form of glucose should be taken if which symptoms develop? Select all that apply. 1. Polyuria 2. Shakiness 3. Palpitations 4. Blurred vision 5. Lightheadedness 6. Fruity breath odor

2. Shakiness 3. Palpitations 5. Lightheadedness

A newly diagnosed client with diabetes mellitus is started on a two-dose insulin protocol combination of short- and intermediate-acting insulin injected twice daily. What portion of the total dose is given before breakfast and what portion before the evening meal? 1. Half before breakfast and half before the evening meal 2. Two thirds before breakfast and one third before the evening meal 3. One third before breakfast and two thirds before the evening meal 4. Three fourths before breakfast and one fourth before the evening meal

2. Two thirds before breakfast and one third before the evening meal

The nurse is caring for a client who is scheduled to have a thyroidectomy and provides instructions to the client about the surgical procedure. Which client statement indicates an understanding of the nurse's instructions? 1. "I expect to experience some tingling of my toes, fingers, and lips after surgery." 2. "I will definitely have to continue taking antithyroid medications after this surgery." 3. "I need to place my hands behind my neck when I have to cough or change positions." 4. "I need to turn my head and neck front, back, and laterally every hour for the first 12 hours after surgery."

3. "I need to place my hands behind my neck when I have to cough or change positions."

A client with type 1 diabetes mellitus calls the nurse to report recurrent episodes of hypoglycemia with exercising. Which statement by the client indicates an inadequate understanding of the peak action of NPH insulin and exercise? 1. "The best time for me to exercise is after I eat." 2. "The best time for me to exercise is after breakfast." 3. "The best time for me to exercise is mid- to late afternoon." 4. "The best time for me to exercise is after my morning snack."

3. "The best time for me to exercise is mid- to late afternoon."

A home health nurse is visiting a client with type 1 diabetes mellitus. The client tells the nurse that he is not feeling well and has had a "respiratory infection" for the past week, which seems to be getting worse. After interviewing the client, what should be the initial nursing action? 1. Notify the health care provider. 2. Document the assessment data. 3. Check the client's blood glucose. 4. Obtain the client's sputum for culture and sensitivity.

3. Check the client's blood glucose.

A client with a diagnosis of diabetic ketoacidosis (DKA) is being treated in the emergency department. Which findings would the nurse expect to note as confirming this diagnosis? Select all that apply. 1. Increase in pH 2. Comatose state 3. Deep, rapid breathing 4. Decreased urine output 5. Elevated blood glucose level 6. Low plasma bicarbonate level

3. Deep, rapid breathing 5. Elevated blood glucose level 6. Low plasma bicarbonate level

The nurse is preparing a client with a new diagnosis of hypothyroidism for discharge. The nurse determines that the client understands discharge instructions if the client states that which symptoms are associated with this diagnosis? Select all that apply. 1. Tremors 2. Weight loss 3. Feeling cold 4. Loss of body hair 5. Persistent lethargy 6. Puffiness of the face

3. Feeling cold 4. Loss of body hair 5. Persistent lethargy 6. Puffiness of the face

A nurse is caring for a client with a dysfunctional thyroid gland and is concerned that the client will exhibit signs of thyroid storm. Which is an early indicator of this complication? 1. Constipation 2. Bradycardia 3. Hyperreflexia 4. Low-grade temperature

3. Hyperreflexia

A hospitalized client is experiencing an episode of hypoglycemia. The nurse plans care, knowing that which is the physiological mechanism that should take place to combat the decline in the blood glucose level? 1. Decreased cortisol release 2. Increased insulin secretion 3. Increased glucagon secretion 4. Decreased epinephrine release

3. Increased glucagon secretion

The emergency department nurse is preparing a plan for initial care of a client with a diagnosis of hyperglycemic hyperosmolar state (HHS). The nurse understands that the hyperglycemia associated with this disorder results from which occurrence? 1. Increased use of glucose 2. Overproduction of insulin 3. Increased production of glucose 4. Increased osmotic movement of water

3. Increased production of glucose

A client's serum blood glucose level is 389 mg/dL. The nurse would expect to note which as an additional finding when assessing this client? 1. Unsteady gait 2. Slurred speech 3. Increased thirst 4. Cold, clammy skin

3. Increased thirst

The nurse is providing instructions regarding insulin administration for a client newly diagnosed with diabetes mellitus. The health care provider has prescribed a mixture of Humulin N and Humulin R insulin. The nurse should instruct the client that which is thefirst step in this procedure? 1. Draw up the correct dosage of Humulin N insulin into the syringe. 2. Draw up the correct dosage of Humulin R insulin into the syringe. 3. Inject air equal to the amount of Humulin N prescribed into the vial of Humulin N insulin. 4. Inject air equal to the amount of Humulin R prescribed into the vial of Humulin R insulin.

3. Inject air equal to the amount of Humulin N prescribed into the vial of Humulin N insulin.

The nurse should include which interventions in the plan of care for a client with hypothyroidism? Select all that apply. 1. Provide a cool environment for the client. 2. Instruct the client to consume a high-fat diet. 3. Instruct the client about thyroid replacement therapy. 4. Encourage the client to consume fluids and high-fiber foods in the diet. 5. Inform the client that iodine preparations will be prescribed to treat the disorder. 6. Instruct the client to contact the health care provider (HCP) if episodes of chest pain occur.

3. Instruct the client about thyroid replacement therapy. 4. Encourage the client to consume fluids and high-fiber foods in the diet. 6. Instruct the client to contact the health care provider (HCP) if episodes of chest pain occur.

A client is admitted to a hospital with a diagnosis of diabetic ketoacidosis (DKA). The initial blood glucose level was 950 mg/dL. A continuous intravenous infusion of short-acting insulin is initiated, along with intravenous rehydration with normal saline. The serum glucose level is now 240 mg/dL. The nurse would next prepare to administer which item? 1. Ampule of 50% dextrose 2. NPH insulin subcutaneously 3. Intravenous fluids containing dextrose 4. Phenytoin (Dilantin) for the prevention of seizures

3. Intravenous fluids containing dextrose

A client is brought to the emergency department in an unresponsive state, and a diagnosis of hyperglycemic hyperosmolar state (HHS) is made. The nurse would immediately prepare to initiate which anticipated health care provider's prescription? 1. Endotracheal intubation 2. 100 units of NPH insulin 3. Intravenous infusion of normal saline 4. Intravenous infusion of sodium bicarbonate

3. Intravenous infusion of normal saline

The nurse is caring for a client after thyroidectomy. The client expresses concern about the postoperative voice hoarseness she is experiencing and asks if the hoarseness will subside. The nurse should provide the client with which information? 1. The hoarseness is permanent. 2. It indicates nerve damage. 3. It is normal during this time and will subside. 4. It will worsen before it subsides, which may take 6 months.

3. It is normal during this time and will subside.

A nurse is assisting a client with diabetes mellitus who is recovering from diabetic ketoacidosis (DKA) to develop a plan to prevent a recurrence. Which is most important to include in the plan of care? 1. Test urine for ketone levels. 2. Eat six small meals per day. 3. Monitor blood glucose levels frequently. 4. Receive appropriate follow-up health care.

3. Monitor blood glucose levels frequently.

The nurse is preparing to care for a client after parathyroidectomy. The nurse should plan for which action for this client? 1. Maintain an endotracheal tube for 24 hours. 2. Administer a continuous mist of room air or oxygen. 3. Place in a flat position with the head and neck immobilized. 4. Use only a rectal thermometer for temperature measurement.

3. Place in a flat position with the head and neck immobilized.

A client has just been admitted to the nursing unit following thyroidectomy. Which assessment is the priority for this client? 1. Hypoglycemia 2. Level of hoarseness 3. Respiratory distress 4. Edema at the surgical site

3. Respiratory distress

A client with type 1 diabetes mellitus is to begin an exercise program, and the nurse is providing instructions regarding the program. Which instruction should the nurse include in the teaching plan? 1. Try to exercise before mealtimes. 2. Administer insulin after exercising. 3. Take a blood glucose test before exercising. 4. Exercise is best performed during peak times of insulin.

3. Take a blood glucose test before exercising.

The nurse performs a physical assessment on a client with type 2 diabetes mellitus. Findings include a fasting blood glucose level of 120 mg/dL, temperature of 101° F, pulse of 88 beats/minute, respirations of 22 breaths/minute, and blood pressure of 100/72 mm Hg. Which assessment would be of most concern to the nurse? 1. Pulse 2. Respiration 3. Temperature 4. Blood pressure

3. Temperature

The nurse is monitoring a client for signs of hypocalcemia after thyroidectomy. Which sign/symptom, if noted in the client, wouldmost likely indicate the presence of hypocalcemia? 1. Bradycardia 2. Flaccid paralysis 3. Tingling around the mouth 4. Absence of Chvostek's sign

3. Tingling around the mouth

The nurse is caring for a client after thyroidectomy. The nurse notes that calcium gluconate is prescribed for the client. The nurse determines that this medication has been prescribed for which purpose? 1. To treat thyroid storm 2. To prevent cardiac irritability 3. To treat hypocalcemic tetany 4. To stimulate release of parathyroid hormone

3. To treat hypocalcemic tetany

The nurse has provided dietary instructions to a client with a diagnosis of hypoparathyroidism. The nurse should instruct the client that it is acceptable to include which item in the diet? 1. Fish 2. Cereals 3. Vegetables 4. Meat and poultry

3. Vegetables

A multidisciplinary health care team is developing a plan of care for a client with hyperparathyroidism. The nurse should include which priority intervention in the plan of care? 1. Restrict fluids to 1000 mL per day. 2. Describe the use of loperamide (Imodium). 3. Walk down the hall for 15 minutes three times a day. 4. Describe the administration of aluminum hydroxide gel.

3. Walk down the hall for 15 minutes three times a day.

The home care nurse visits a client with a diagnosis of hyperparathyroidism who is taking furosemide (Lasix) and provides dietary instructions to the client. Which statement by the client indicates a need for additional instruction? 1. "I need to eat foods high in potassium." 2. "I need to drink at least 2 to 3 L of fluid daily." 3. "I need to eat small, frequent meals and snacks if nauseated." 4. "I need to increase my intake of dietary items that are high in calcium."

4. "I need to increase my intake of dietary items that are high in calcium."

The nurse provides dietary instructions to a client with diabetes mellitus regarding the prescribed diet. Which statement, if made by the client, indicates a need for further teaching? 1. "I'll eat a balanced meal plan." 2. "I need to drink diet soft drinks." 3. "I'll snack on fruit instead of cake." 4. "I need to purchase special dietetic foods."

4. "I need to purchase special dietetic foods."

The nurse has provided instructions for measuring blood glucose levels to a client newly diagnosed with diabetes mellitus who will be taking insulin. The client demonstrates understanding of the instructions by identifying which method as the best method for monitoring blood glucose levels? 1. "I will check my blood glucose level every day at 5:00 pm." 2. "I will check my blood glucose level 1 hour after each meal." 3. "I will check my blood glucose level 2 hours after each meal." 4. "I will check my blood glucose level before each meal and at bedtime."

4. "I will check my blood glucose level before each meal and at bedtime.

The nurse provides instructions to a client newly diagnosed with type 1 diabetes mellitus. The nurse recognizes accurate understanding of measures to prevent diabetic ketoacidosis when the client makes which statement? 1. "I will stop taking my insulin if I'm too sick to eat." 2. "I will decrease my insulin dose during times of illness." 3. "I will adjust my insulin dose according to the level of glucose in my urine." 4. "I will notify my health care provider (HCP) if my blood glucose level is higher than 250 mg/dL."

4. "I will notify my health care provider (HCP) if my blood glucose level is higher than 250 mg/dL."

A client with type 1 diabetes mellitus is having trouble remembering the types, duration, and onset of the action of insulin. The client tells the nurse that the family members have not been supportive. Which response by the nurse is best? 1. "What is it that you don't understand?" 2. "You can't always depend on your family to help." 3. "It's not really necessary for you to remember this." 4. "Let me go over the types of insulin with you again."

4. "Let me go over the types of insulin with you again."

A client with diabetes mellitus has been instructed in the dietary exchange system. The client asks the nurse if bacon is allowed in the diet. Which nursing response is most appropriate? 1. "Bacon is not allowed." 2. "Bacon is much too high in fat." 3. "Bacon may be eaten if you eliminate one meat item from your diet." 4. "One strip of bacon may be eaten if you eliminate 1 teaspoon of butter."

4. "One strip of bacon may be eaten if you eliminate 1 teaspoon of butter."

The nurse is interviewing a client with type 2 diabetes mellitus. Which statement by the client indicates an understanding of the treatment for this disorder? 1. "I take oral insulin instead of shots." 2. "By taking these medications, I am able to eat more." 3. "When I become ill, I need to increase the number of pills I take." 4. "The medications I'm taking help release the insulin I already make."

4. "The medications I'm taking help release the insulin I already make."

A client is undergoing an oral glucose tolerance test. The nurse interprets that the client's results are compatible with diabetes mellitus if the glucose level is at which value after 120 minutes (2 hours)? 1. 80 mg/dL 2. 110 mg/dL 3. 130 mg/dL 4. 160 mg/dL

4. 160 mg/dL

The nurse is caring for a client admitted to the emergency department with diabetic ketoacidosis (DKA). In the acute phase, the nurse plans for which priority intervention? 1. Correct the acidosis. 2. Administer 5% dextrose intravenously. 3. Apply a monitor for an electrocardiogram. 4. Administer short-duration insulin intravenously.

4. Administer short-duration insulin intravenously.

The nurse is reviewing the health care provider (HCP) prescriptions for a client with a diagnosis of diabetes mellitus who has been hospitalized for treatment of an infected foot ulcer. The nurse expects to note which finding in the HCP prescriptions? 1. A decreased-calorie diet 2. An increased-calorie diet 3. A decreased amount of NPH daily insulin 4. An increased amount of NPH daily insulin

4. An increased amount of NPH daily insulin

The nurse is developing a plan of care for a client who is scheduled for a thyroidectomy. The nurse focuses on psychosocial needs, knowing that which is likely to occur in the client? 1. Infertility 2. Gynecomastia 3. Sexual dysfunction 4. Body image changes

4. Body image changes

An external insulin pump is prescribed for a client with diabetes mellitus and the client asks the nurse about the functioning of the pump. The nurse bases the response on which information about the pump? 1. Is timed to release programmed doses of short-duration or NPH insulin into the bloodstream at specific intervals 2. Continuously infuses small amounts of NPH insulin into the bloodstream while regularly monitoring blood glucose levels 3. Is surgically attached to the pancreas and infuses regular insulin into the pancreas, which in turn releases the insulin into the bloodstream 4. Gives a small continuous dose of short-duration insulin subcutaneously, and the client can self-administer a bolus with an additional dose from the pump before each meal

4. Gives a small continuous dose of short-duration insulin subcutaneously, and the client can self-administer a bolus with an additional dose from the pump before each meal

A client arrives in the hospital emergency department complaining of severe thirst and polyuria. The client tells the nurse that she has a history of diabetes mellitus. A blood glucose level is drawn, and the result is 685 mg/dL. Which intervention should the nurse anticipate to be initially prescribed for the client? 1. Glucagon via the subcutaneous route 2. Glyburide (DiaBeta) via the oral route 3. Humulin N insulin via the subcutaneous route 4. Humulin R insulin via the intravenous (IV) route

4. Humulin R insulin via the intravenous (IV) route

A client newly diagnosed with diabetes mellitus has been stabilized with daily insulin injections. A nurse prepares a discharge teaching plan regarding the insulin and plans to reinforce which concept? 1. Always keep insulin vials refrigerated. 2. Ketones in the urine signify a need for less insulin. 3. Increase the amount of insulin before excessive exercise. 4. Systematically rotate insulin injections within one anatomical site.

4. Systematically rotate insulin injections within one anatomical site.

The nurse is assessing the learning readiness of a client newly diagnosed with diabetes mellitus. Which behavior indicates to the nurse that the client is not ready to learn? 1. The client asks if the spouse may attend the teaching session. 2. The client asks appropriate questions about what will be taught. 3. The client asks for written materials about diabetes mellitus before class. 4. The client complains of fatigue whenever the nurse plans a teaching session.

4. The client complains of fatigue whenever the nurse plans a teaching session.

ANS: D BCC is frequently associated with sun exposure and preventive measures should be taken for future sun exposure. BCC spreads locally, and does not metastasize to distant tissues. Since BCC can cause local tissue destruction, treatment is indicated. Local (not systemic) chemotherapy may be used to treat BCC.

A nurse develops a teaching plan for a patient diagnosed with basal cell carcinoma (BCC). Which information should the nurse include in the teaching plan? a. Treatment plans include watchful waiting. b. Screening for metastasis will be important. c. Low dose systemic chemotherapy is used to treat BCC. d. Minimizing sun exposure will reduce risk for future BCC.

ANS: A, B, E Cool or tepid baths, cool dressings, and OTC antihistamines all help reduce pruritus and scratching. Adding oil to bath water is not recommended because of the increased risk for falls. The patient should use the towel to pat (not rub) the skin dry.

A nurse is teaching a patient with contact dermatitis of the arms and legs about ways to decrease pruritus. Which information should the nurse include in the teaching plan (select all that apply)? a. Cool, wet cloths or dressings can be used to reduce itching. b. Take cool or tepid baths several times daily to decrease itching. c. Add oil to your bath water to aid in moisturizing the affected skin. d. Rub yourself dry with a towel after bathing to prevent skin maceration. e. Use of an over-the-counter (OTC) antihistamine can reduce scratching.

ANS: D Because the only risk factor that the patient can change is the use of a tanning booth, the nurse should focus teaching about melanoma prevention on this factor. The other factors also will contribute to increased risk for melanoma.

A patient has the following risk factors for melanoma. Which risk factor should the nurse assign as the priority focus of patient teaching? a. The patient has multiple dysplastic nevi. b. The patient is fair-skinned and has blue eyes. c. The patient's mother died of a malignant melanoma. d. The patient uses a tanning booth throughout the winter.

ANS: A Because the appearance of the lesion suggests actinic keratosis or possible squamous cell carcinoma (SCC), the appropriate treatment would be excision and biopsy. Over-the-counter (OTC) corticosteroids, topical antibiotics, and Retin-A would not be used for this lesion.

A patient in the dermatology clinic has a thin, scaly erythematous plaque on the right cheek. Which action should the nurse take? a. Prepare the patient for a biopsy. b. Teach about the use of corticosteroid creams. c. Explain how to apply tretinoin (Retin-A) to the face. d. Discuss the need for topical application of antibiotics.

ANS: C The eyes should be shielded from UV light (UVL) during and after PUVA therapy to prevent the development of cataracts. The patient should be taught about the effects of UVL on unaffected skin, but lead-lined drapes, use of antiseptic soap, and petroleum jelly are not used to prevent skin damage.

A patient is undergoing psoralen plus ultraviolet A light (PUVA) therapy for treatment of psoriasis. What action should the nurse take to prevent adverse effects from this procedure? a. Cleanse the skin carefully with an antiseptic soap. b. Shield any unaffected areas with lead-lined drapes. c. Have the patient use protective eyewear while receiving PUVA. d. Apply petroleum jelly to the areas surrounding the psoriatic lesions.

ANS: D The nurse's initial actions should be to assess the impact of the disease on the patient's life and to allow the patient to verbalize feelings about the psoriasis. Depending on the assessment findings, other actions may be appropriate.

A patient who has severe refractory psoriasis on the face, neck, and extremities is socially withdrawn because of the appearance of the lesions. Which action should the nurse take first? a. Discuss the possibility of enrolling in a worker-retraining program. b. Encourage the patient to volunteer to work on community projects. c. Suggest that the patient use cosmetics to cover the psoriatic lesions. d. Ask the patient to describe the impact of psoriasis on quality of life.

ANS: D The description of the mole is consistent with malignancy, so excision and biopsy are indicated. Curettage and cryosurgery are not used if malignancy is suspected. A punch biopsy would not be done for a lesion greater than 5 mm in diameter.

A patient with an enlarging, irregular mole that is 7 mm in diameter is scheduled for outpatient treatment. The nurse should plan to prepare the patient for which procedure? a. Curettage b. Cryosurgery c. Punch biopsy d. Surgical excision

ANS: D The patient should be taught that transient burning at the application site is an expected effect of pimecrolimus and that the medication should be left in place. The other statements by the patient are accurate and indicate that patient teaching has been effective.

A patient with atopic dermatitis has a new prescription for pimecrolimus (Elidel). After teaching the patient about the medication, which statement by the patient indicates that further teaching is needed? a. "After I apply the medication, I can go ahead and get dressed as usual." b. "I will need to minimize my time in the sun while I am using the Elidel." c. "I will rub the medication gently onto the skin every morning and night." d. "If the medication burns when I apply it, I will wipe it off and call the doctor."

ANS: A Thinning of the skin indicates that atrophy, a possible adverse effect of topical corticosteroids, is occurring. The health care provider should be notified so that the medication can be changed or tapered. Alopecia, red-brown discoloration, and dryness/scaling of the skin are not adverse effects of topical corticosteroid use.

A patient with atopic dermatitis has been using a high-potency topical corticosteroid ointment for several weeks. The nurse should assess for which adverse effect? a. Thinning of the affected skin b. Alopecia of the affected areas c. Reddish-brown discoloration of the skin d. Dryness and scaling in the areas of treatment

A 72-year-old patient was admitted with epigastric pain due to a gastric ulcer. Which patient assessment warrants an urgent change in the nursing plan of care? A) Chest pain relieved with eating or drinking water B) Back pain 3 or 4 hours after eating a meal C) Burning epigastric pain 90 minutes after breakfast D) Rigid abdomen and vomiting following indigestion

A rigid abdomen with vomiting in a patient who has a gastric ulcer indicates a perforation of the ulcer, especially if the manifestations of perforation appear suddenly. Midepigastric pain is relieved by eating, drinking water, or antacids with duodenal ulcers, not gastric ulcers. Back pain 3-4 hours after a meal is more likely to occur with a duodenal ulcer. Burning epigastric pain 1-2 hours after a meal is an expected manifestation of a gastric ulcer related to increased gastric secretions and does not cause an urgent change in the nursing plan of care.

ANS: B Pediculosis is characterized by wheal-like lesions with parasites that attach eggs to the base of the hair shaft. The other descriptions are more characteristic of other types of skin disorders.

A teenaged male patient who wrestles in high school is examined by the nurse in the clinic. Which assessment finding would prompt the nurse to teach the patient about the importance of not sharing headgear to prevent the spread of pediculosis? a. Ringlike rashes with red, scaly borders over the entire scalp b. Papular, wheal-like lesions with white deposits on the hair shaft c. Patchy areas of alopecia with small vesicles and excoriated areas d. Red, hivelike papules and plaques with sharply circumscribed borders

When preparing a patient for a capsule endoscopy study, what should the nurse do? A) Ensure the patient understands the required bowel preparation. B) Have the patient return to the procedure room for removal of the capsule. C) Teach the patient to maintain a clear liquid diet throughout the procedure. D) Explain to the patient that conscious sedation will be used during placement of the capsule.

A) A capsule endoscopy study involves the patient performing a bowel prep to cleanse the bowel before swallowing the capsule. The patient will be on a clear liquid diet for 1 to 2 days before the procedure and will remain NPO for 4 to 6 hours after swallowing the capsule. The capsule is disposable and will pass naturally with the bowel movement, although the monitoring device will need to be removed.

After administering a dose of promethazine (Phenergan) to a patient with nausea and vomiting, the nurse explains that which of the following may be experienced as a common temporary adverse effect of the medication? A) Drowsiness B) Reduced hearing C) Sensation of falling D) Photosensitivity

A) Drowsiness (Although being given to this patient as an antiemetic, promethazine also has sedative and amnesic properties. For this reason, the patient is likely to experience drowsiness as an adverse effect of the medication.)

A patient had a stomach resection for stomach cancer. The nurse should teach the patient about the loss of the hormone that stimulates gastric acid secretion and motility and maintains lower esophageal sphincter tone. Which hormone will be decreased with a gastric resection? A) Gastrin B) Secretin C) Cholecystokinin D) Gastric inhibitory peptide

A) Gastrin is the hormone activated in the stomach (and duodenal mucosa) by stomach distention that stimulates gastric acid secretion and motility and maintains lower esophageal sphincter tone. Secretin inhibits gastric motility and acid secretion and stimulates pancreatic bicarbonate secretion. Cholecystokinin allows increased flow of bile into the duodenum and release of pancreatic digestive enzymes. Gastric inhibitory peptide inhibits gastric acid secretion and motility.

The patient with cirrhosis is being taught self-care. Which statement indicates the patient needs more teaching? A) "If I notice a fast heart rate or irregular beats, this is normal for cirrhosis." B) "I need to take good care of my belly and ankle skin where it is swollen." C) "A scrotal support may be more comfortable when I have scrotal edema." D) "I can use pillows to support my head to help me breathe when I am in bed."

A) If the patient with cirrhosis experiences a fast or irregular heart rate, it may be indicative of hypokalemia and should be reported to the health care provider, as this is not normal for cirrhosis. Edematous tissue is subject to breakdown and needs meticulous skin care. Pillows and a semi-Fowler's or Fowler's position will increase respiratory efficiency. A scrotal support may improve comfort if there is scrotal edema

The wound, ostomy, and continence (WOC) nurse selects the site where the ostomy will be placed. What should be included in the consideration for the site? A) The patient must be able to see the site. B) Outside the rectus muscle area is the best site. C) It is easier to seal the drainage bag to a protruding area. D) The ostomy will need irrigation, so area should not be tender.

A) In selection of the ostomy site, the WOC nurse will want a site visible to the patient so the patient can take care of it, within the rectus muscle to avoid hernias, and on a flat surface to more easily create a good seal with the drainage bag.

3. A nurse is reviewing the health record of a client who has syndrome of inappropriate antidiuretic hormone (SIADH). Which of the following laboratory findings should the nurse anticipate? (Select all that apply.) A. Low serum sodium B. High serum potassium C. Decreased urine osmolality D. High urine sodium E. Increased urine-specific gravity

A. Low serum sodium D. High urine sodium E. Increased urine-specific gravity

The nurse is conducting discharge teaching for a patient with metastatic lung cancer who was admitted with a bowel impaction. Which of the following instructions would be most helpful to prevent further episodes of constipation? A) Maintain a high intake of fluid and fiber in the diet. B) Reduce intake of medications causing constipation. C) Eat several small meals per day to maintain bowel motility. D) Sit upright during meals to increase bowel motility by gravity.

A) Increased fluid intake and a high-fiber diet reduce the incidence of constipation caused by immobility, medications, and other factors. Fluid and fiber provide bulk that in turn increases peristalsis and bowel motility. Analgesics taken for lung cancer probably cannot be reduced. Other medications may decrease constipation, but it is best to avoid laxatives. Eating several small meals per day and position do not facilitate bowel motility. Defecation is easiest when the person sits on the commode with the knees higher than the hips.

A stroke patient who primarily uses a wheelchair for mobility has diarrhea with fecal incontinence. What should the nurse assess first? A) Fecal impaction B) Perineal hygiene C) Dietary fiber intake D) Antidiarrheal agent use

A) Patients with limited mobility are at risk for fecal impactions due to constipation that may lead to liquid stool leaking around the hardened impacted feces, so assessing for fecal impaction is the priority. Perineal hygiene can be assessed at the same time. Assessing the dietary fiber and fluid intake and antidiarrheal agent use will be assessed and considered next.

A pt with a history of peptic ulcer disease has presented to the emergency department with complaints of severe abdominal pain and a rigid, boardlike abdomen, prompting the health care team to suspect a perforated ulcer. Which of the following actions should the nurse anticipate? A) Providing IV fluids and inserting a nasogastric tube B) Administering oral bicarbonate and testing the patient's gastric pH level C) Performing a fecal occult blood test and administering IV calcium gluconate D) Starting parenteral nutrition and placing the patient in a high-Fowler's position

A) Providing IV fluids and inserting a nasogastric tube A perforated peptic ulcer requires IV replacement of fluid losses and continued gastric aspiration by NG tube. Nothing is given by mouth and gastric pH testing is not a priority. Calcium gluconate is not a medication directly relevant to the patient's suspected diagnosis and parenteral nutrition is not a priority in the short term.

The nurse is preparing to insert a nasogastric (NG) tube into a 68-year-old female patient who is nauseated and vomiting. She has an abdominal mass and suspected small intestinal obstruction. The patient asks the nurse why this procedure is necessary. What response by the nurse is most appropriate? A) "The tube will help to drain the stomach contents and prevent further vomiting." B) "The tube will push past the area that is blocked and thus help to stop the vomiting." C) "The tube is just a standard procedure before many types of surgery to the abdomen." D) "The tube will let us measure your stomach contents so that we can plan what type of IV fluid replacement would be best."

A) The NG tube is used to decompress the stomach by draining stomach contents and thereby prevent further vomiting. The NG tube will not push past the blocked area. Potential surgery is not currently indicated. The location of the obstruction will determine the type of fluid to use, not measure the amount of stomach contents.

When evaluating the patient's understanding about the care of the ileostomy, what statement by the patient indicates the patient needs more teaching? A) "I will be able to regulate when I have stools." B) "I will be able to wear the pouch until it leaks." C) "Dried fruit and popcorn must be chewed very well." D) "The drainage from my stoma can damage my skin."

A) The ileostomy is in the ileum and drains liquid stool frequently, unlike the colostomy which has more formed stool the further distal the ostomy is in the colon. The ileostomy pouch is usually worn 4-7 days or until it leaks. It must be changed immediately if it leaks because the drainage is very irritating to the skin. To avoid obstruction, popcorn, dried fruit, coconut, mushrooms, olives, stringy vegetables, food with skin, and meats with casings must be chewed extremely well before swallowing because of the narrow diameter of the ileostomy lumen.

A patient with type 2 diabetes and cirrhosis asks the nurse if it would be okay to take silymarin (milk thistle) to help minimize liver damage. The nurse responds based on knowledge that: A. Milk thistle may affect liver enzymes and thus alter drug metabolism. B. Milk thistle is generally safe in recommended doses for up to 10 years. C. is unclear scientific evidence for the use of milk thistle in treating cirrhosis. D. Milk thistle may elevate the serum glucose levels and is thus contraindicated in diabetes.

A) There is good scientific evidence that there is no real benefit from using milk thistle to protect the liver cells from toxic damage in the treatment of cirrhosis. Milk thistle does affect liver enzymes and thus could alter drug metabolism. Therefore patients will need to be monitored for drug interactions. It is noted to be safe for up to 6 years, not 10 years, and it may lower, not elevate, blood glucose levels.

When administered a dose of metoclopramide (Reglan), a patient complains of nausea. The nurse would teach the patient to report which of the following potential adverse effects? A) Tremors B) Constipation C) Double vision D) Numbness in the fingers and dose

A) Tremors (Extrapyramidal side effects, including tremors and dyskinesias, may occur as a result of metoclopramide (Reglan) administration.)

Two days following a colectomy for an abdominal mass, a patient reports gas pains and abdominal distention. The nurse plans care for the patient based on the knowledge that the symptoms are occurring as a result of A) impaired peristalsis. B) irritation of the bowel. C) nasogastric suctioning. D) inflammation of the incision site.

A) Until peristalsis returns to normal following anesthesia, the patient may experience slowed gastrointestinal motility leading to gas pains and abdominal distention. Irritation of the bowel, nasogastric suctioning, and inflammation of the surgical site do not cause gas pains or abdominal distention.

A 90-year-old healthy man is suffering from dysphagia. The nurse explains what age-related change of the GI tract is the most likely cause of his difficulty? A) Xerostomia B) Esophageal cancer C) Decreased taste buds D) Thinner abdominal wall

A) Xerostomia, decreased volume of saliva, leads to dry oral mucosa and dysphagia. Esophageal cancer is not an age-related change. Decreased taste buds and a thinner abdominal wall do not contribute to difficulty swallowing.

4. A nurse is assessing a client who has SIADH. Which of the following findings indicate the client is experiencing a complication? A. Decreased central venous pressure (CVP) B. Increased urine output C. Distended neck veins D. Extreme thirst

C. Distended neck veins

When caring for a patient with a biliary obstruction, the nurse will anticipate administering which of the following vitamin supplements (select all that apply)? A. Vitamin A B. Vitamin D C. Vitamin E D. Vitamin K E. Vitamin B

A,B,C,D) Biliary obstruction prevents bile from entering the small intestine and thus prevents the absorption of fat-soluble vitamins. Vitamins A, D, E, and K are all fat-soluble and thus would need to be supplemented in a patient with biliary obstruction.

When caring for a patient with liver disease, the nurse recognizes the need to prevent bleeding resulting from altered clotting factors and rupture of varices. Which of the following nursing interventions would be appropriate to achieve this outcome (select all that apply)? A. Use smallest gauge possible when giving injections or drawing blood. B. Teach patient to avoid straining at stool, vigorous blowing of nose, and coughing. C. Advise patient to use soft-bristle toothbrush and avoid ingestion of irritating food. D. Apply gentle pressure for the shortest possible time period after performing venipuncture. E. Instruct patient to avoid aspirin and NSAIDs to prevent hemorrhage when varices are present.

A,B,C,E) Using the smallest gauge needle for injections will minimize the risk of bleeding into the tissues. Avoiding straining, nose blowing, and coughing will reduce the risk of hemorrhage at these sites. The use of a soft-bristle toothbrush and avoidance of irritating food will reduce injury to highly vascular mucous membranes. The nurse should apply gentle but prolonged pressure to venipuncture sites to minimize the risk of bleeding. Aspirin and NSAIDs should not be used in patients with liver disease because they interfere with platelet aggregation, thus increasing the risk for bleeding.

The patient with cirrhosis has an increased abdominal girth from ascites. The nurse should know that this fluid gathers in the abdomen for which reasons (select all that apply)? A) There is decreased colloid oncotic pressure from the liver's inability to synthesize albumin. B) Hyperaldosteronism related to damaged hepatocytes increases sodium and fluid retention. C) Portal hypertension pushes proteins from the blood vessels, causing leaking into the peritoneal cavity. D) Osmoreceptors in the hypothalamus stimulate thirst, which causes the stimulation to take in fluids orally. E) Overactivity of the enlarged spleen results in increased removal of blood cells from the circulation, which decreases the vascular pressure.

A,B.C) The ascites related to cirrhosis are caused by decreased colloid oncotic pressure from the lack of albumin from liver inability to synthesize it and the portal hypertension that shifts the protein from the blood vessels to the peritoneal cavity, and hyperaldosteronism which increases sodium and fluid retention. The intake of fluids orally and the removal of blood cells by the spleen do not directly contribute to ascites.

5. A nurse is providing teaching to a client who has a new diagnosis of diabetes insipidus. Which of the following statements by the client requires further teaching? A. "I can drink up to 2 quarts of fluid a day." B. "I should expect to urinate frequently at night." C. "I may experience headaches." D. "I may experience a dry mouth."

A. "I can drink up to 2 quarts of fluid a day."

Which patient would be more likely to have the highest risk of developing malignant melanoma? A. A fair-skinned woman who uses a tanning booth regularly B. An African American patient with a family history of cancer C. An adult who required phototherapy as an infant for the treatment of hyperbilirubinemia D. A Hispanic male with a history of psoriasis and eczema that responded poorly to treatment

A. A fair-skinned woman who uses a tanning booth regularly Risk factors for malignant melanoma include a fair complexion and exposure to ultraviolet light. Psoriasis, eczema, short-duration phototherapy, and a family history of other cancers are less likely to be linked to malignant melanoma.

Which nursing intervention would be most helpful in managing a patient newly admitted with cellulitis of the right foot? A. Applying warm, moist heat B. Wrapping the foot snugly in blankets C. Limiting ambulation to three times daily D. Keeping the foot at or below heart level

A. Applying warm moist heat The application of warm, moist heat speeds the resolution of inflammation and infection when accompanied by appropriate antibiotic therapy. It does this by increasing local circulation to the affected area to bring macrophages to the area and carry off cellular debris. Immobilization and elevation is also used. Snug blankets would not be helpful and could decrease circulation to this sensitive tissue.

4. A nurse is caring for a client who has primary adrenal insufficiency. Which of the following findings should the nurse anticipate after an IV injection of ACTH 1.0 mg? A. Decrease in serum plasma cortisol B. Elevated fasting serum blood glucose C. Decrease in serum sodium D. Increase in urinary output

A. Decrease in serum plasma cortisol

3. A nurse is caring for a client who has syndrome of inappropriate antidiuretic hormone (SIADH). Which of the following findings should the nurse expect? (Select all that apply.) A. Decreased serum sodium B. Urine specific gravity 1.001 C. Serum osmolarity 230 mOsm/L D. Polyuria E. Increased thirst

A. Decreased serum sodium C. Serum osmolarity 230 mOsm/L

5. A nurse is providing discharge teaching to a client who experienced diabetic ketoacidosis. Which of the following should the nurse include in the teaching? (Select all that apply.) A. Drink 3 L of fluids daily. B. Monitor blood glucose every 4 hr when ill. C. Administer insulin as prescribed when ill. D. Notify the provider when blood glucose is 200 mg/dL. E. Report ketones in the urine after 24 hr of illness

A. Drink 3 L of fluids daily. B. Monitor blood glucose every 4 hr when ill. C. Administer insulin as prescribed when ill. E. Report ketones in the urine after 24 hr of illness

4. A nurse is presenting information to a group of clients about nutrition habits that prevent type 2 diabetes mellitus. Which of the following should the nurse include in the information? (Select all that apply.) A. Eat less meat and processed foods. B. Decrease intake of saturated fats. C. Increase daily fiber intake. D. Limit saturated fat intake to 15% of daily caloric intake. E. Include omega-3 fatty acids in the diet.

A. Eat less meat and processed foods. B. Decrease intake of saturated fats. C. Increase daily fiber intake. E. Include omega-3 fatty acids in the diet.

The client is diagnosed with hyperthyroidism. Which of the following clinical manifestations of hyperthyroidism should the nurse expect to find? (select all that apply.) A. Excessive Sweating B. Tremors C. Gastric Hypermotility D. Increased White Blood Cell Count E. Hypothermia F. Photophobia G. Exophthalmus

A. Excessive Sweating B. Tremors C. Gastric Hypermotility F. Photophobia G. Exophthalmus

The patient is in the hospital for a surgical procedure and has dry skin and pruritis on her legs that causes her to scratch at the skin uncontrollably. What measures can the nurse use to help stop the itch/scratch cycle 0?(select all that apply)? A. Moisturize the skin on the legs. B. Provide a warm blanket and room. C. Administer antihistamines at bedtime. D. Use careful hand washing after rubbing her legs. E. Cleanse the legs with a saline solution twice daily.

A. Moisturize the skin on the legs. C. Administer antihistamines at bedtime. Moisturizing the skin to decrease the dryness and the itch sensation and bedtime antihistamines to decrease a potential allergic reaction and provide some sedation will help the patient sleep since pruritis is often worse at night and the patient needs sleep for healing. Using nonallergic sheets may also help. Anything causing vasodilation, such as warmth or rubbing, should be avoided. Saline solution would only further dry the skin so would not be used on the patient's legs.

5. A nurse in a provider's office is planning care for a client who has a new diagnosis of Graves' disease and a new prescription for methimazole (Tapazole). Which of the following should the nurse include in the plan of care? (Select all that apply.) A. Monitor CBC. B. Monitor triiodothyronine (T3). C. Inform the client that the medication should not be taken for more than 3 months. D. Advise the client to take the medication at the same time every day. E. Inform the client that an adverse effect of this medication is iodine toxicity.

A. Monitor CBC. B. Monitor triiodothyronine (T3). D. Advise the client to take the medication at the same time every day.

The patient has been diagnosed with tinea unguium (Onychomycosis) under her nails. She does not like the oral antifungal medication. What is the best alternate treatment the nurse should describe for her? A. Nail avulsion B. Antifungal cream C. Thinning of fingernails D. Soaking nails in salt water

A. Nail avulsion Nail avulsion is the best alternate treatment to the oral antifungal medication. Antifungal cream is minimally effective. Thinning fingernails is not needed if the tinea unguium is under her toenails. Soaking the nails will not be helpful.

The patient with a stage IV pressure ulcer on the coccyx will need a skin graft to close the wound. What postoperative care should the nurse expect to use to facilitate healing? A. No straining of the grafted site B. The wound will be exposed to air. C. Soft tissue expansion will be done daily. D. The pressure dressing will not be removed.

A. No straining of the grafted site Straining or stretching of the grafted site must be avoided to allow the graft to be vascularized and fixed to the new site for healing. The wound may or may not be exposed to air depending on the type of graft, and the donor site will be covered with a protective dressing to prevent further damage. Soft tissue expansion and pressure dressings will not be used after this wound's skin graft.

4. A nurse in an intensive care unit is admitting a client who has myxedema coma. Which of the following should the nurse anticipate in caring for this client? (Select all that apply.) A. Observe cardiac monitor for inverted T wave. B. Observe for evidence of urinary tract infection. C. Initiate IV fluids using 0.9% sodium chloride. D. Expect a prescription for levothyroxine (Synthroid) IV bolus. E. Provide warmth using a heating pad.

A. Observe cardiac monitor for inverted T wave. B. Observe for evidence of urinary tract infection. C. Initiate IV fluids using 0.9% sodium chloride. D. Expect a prescription for levothyroxine (Synthroid) IV bolus.

The nurse would assess a patient admitted with cellulitis for what localized manifestation? A. Pain B. Fever C. Chills D. Malaise

A. Pain Pain, redness, heat, and swelling are all localized manifestation of cellulitis. Fever, chills, and malaise are generalized, systemic manifestations of inflammation and infection.

The provider prescribes medication for the client's hyperthyroidism. Which of the following medications blocks the synthesis of thyroid hormone? A. Propylthiouracil (PTU) B. Metoprolol (Lopressor) C. Amoxicillin (Amoxil) D. Docusate sodium (Colace)

A. Propylthiouracil (PTU)

In a patient admitted with cellulitis of the left foot, which clinical manifestation would the nurse expect to find on assessment of the left foot? A. Redness and swelling B. Pallor and poor turgor C. Cyanosis and coolness D. Edema and brown skin discoloration

A. Redness and swelling Cellulitis is a diffuse, acute inflammation of the skin. It is characterized by redness, swelling, heat, and tenderness in the affected area. These changes accompany the processes of inflammation and infection.

1. A nurse is caring for a client who has primary diabetes insipidus. Which of the following manifestations should the nurse expect to find? (Select all that apply.) A. Serum sodium of 155 mEq/L B. Fatigue C. Serum osmolality of 250 mOsm/L D. Polyuria E. Nocturia

A. Serum sodium of 155 mEq/L B. Fatigue D. Polyuria E. Nocturia

4. A nurse is preparing to receive a client from the PACU who is postoperative following a thyroidectomy. The nurse should ensure that which of the following equipment is available? (Select all that apply.) A. Suction equipment B. Humidified air C. Flashlight D. Tracheostomy tray E. Oxygen delivery equipment

A. Suction equipment B. Humidified air D. Tracheostomy tray E. Oxygen delivery equipment

The nurse should teach a patient who is taking which drug to avoid prolonged sun exposure? A. Tetracycline B. Ipratropium C. Morphine sulfate D. Oral contraceptives

A. Tetracycline Several antibiotics, including tetracycline, may cause photosensitivity. This is not the case with ipratropium, morphine, or oral contraceptives.

A 67-year-old woman admitted with heart failure is also diagnosed with herpes zoster (shingles) and draining vesicles. Which action, if observed by the nurse, would require additional teaching for that individual? A. The dietitian wears a mask when entering the patient's room. B. The patient keeps the draining vesicles covered with a dressing. C. The student nurse who takes prednisone requests a different patient assignment. D. The nursing assistant washes hands frequently and wears gloves when in the room.

A. The dietitian wears a mask when entering the patient's room. Herpes zoster (shingles) is spread by contact with fluid draining from the vesicles (not by coughing, sneezing, or casual contact). Shingles is not contagious before the vesicles appear or after the vesicles have crusted over. The risk of a person with shingles spreading the virus is low if the rash is covered. Wearing a mask would not prevent the spread of infection. Until the rash develops crusts, the patient should not have contact with an immune compromised person (e.g., a person taking prednisone). Frequent hand washing helps to prevent the spread of varicella zoster virus.

1. A nurse is reviewing the health record of a client who has hyperglycemic-hyperosmolar state (HHS). Which of the following data confirms this diagnosis? (Select all that apply.) A.Evidence of recent myocardial infraction B.BUN 35 mg/dL C.Takes a calcium channel blocker D.Age 77 years E.No insulin production

A.Evidence of recent myocardial infraction B.BUN 35 mg/dL C.Takes a calcium channel blocker D.Age 77 years

2. A nurse is reviewing the laboratory findings of a client who has suspected hyperthyroidism. An elevation of which of the following supports this diagnosis? A.Triodothyronine (T3) B.Vanillylmandelic acid (VMA) C. Adrenocorticotropic hormone (ACTH) D.Glycosylated hemoglobin (HbA1c)

A.Triodothyronine (T3) - T3 increases in a hyperthyroid state

The patient is having a gastroduodenostomy (Billroth I operation) for stomach cancer. What long-term complication is occurring when the patient reports generalized weakness, sweating, palpitations, and dizziness 15 to 30 minutes after eating? A) Malnutrition B) Bile reflux gastritis C) Dumping syndrome D) Postprandial hypoglycemia

After a Billroth I operation, dumping syndrome may occur 15 to 30 minutes after eating because of the hypertonic fluid going to the intestine and additional fluid being drawn into the bowel. Malnutrition may occur but does not cause these symptoms. Bile reflux gastritis cannot happen when the stomach has been removed. Postprandial hypoglycemia occurs with similar symptoms, but 2 hours after eating.

When providing discharge teaching for the patient after a laparoscopic cholecystectomy, what information should the nurse include? A) A lower-fat diet may be better tolerated for several weeks. B) Do not return to work or normal activities for 3 weeks. C) Bile-colored drainage will probably drain from the incision. D) Keep the bandages on and the puncture site dry until it heals.

Although the usual diet can be resumed, a low-fat diet is usually better tolerated for several weeks following surgery. Normal activities can be gradually resumed as the patient tolerates. Bile-colored drainage or pus, redness, swelling, severe pain, and fever may all indicate infection. The bandage may be removed the day after surgery, and the patient can shower.

A nurse is instructing a client with DM I about exercise. What is an appropriate statement by the nurse?

Always where a medical identification tag while exercising

ANS: D Application of cold packs to the incision after the surgery will help decrease bruising and swelling at the site. Since the Mohs procedure results in complete excision of the lesion, topical fluorouracil is not needed after surgery. After the Mohs procedure the edges of the wound can be left open to heal or the edges can be approximated and sutured together. The suture line can be cleaned with tap water. No debridement with wet-to-dry dressings is indicated.

An older adult patient with a squamous cell carcinoma (SCC) on the lower arm has a Mohs procedure in the dermatology clinic. Which nursing action will be included in the postoperative plan of care? a. Describe the use of topical fluorouracil on the incision. b. Teach how to use sterile technique to clean the suture line. c. Schedule daily appointments for wet-to-dry dressing changes. d. Teach about the use of cold packs to reduce bruising and swelling.

18. When providing discharge teaching for the patient after a laparoscopic cholecystectomy, what information should the nurse include? A. A lower-fat diet may be better tolerated for several weeks. B. Do not return to work or normal activities for 3 weeks. C. Bile-colored drainage will probably drain from the incision. D. Keep the bandages on and the puncture site dry until it heals.

Ans: A Although the usual diet can be resumed, a low-fat diet is usually better tolerated for several weeks following surgery. Normal activities can be gradually resumed as the patient tolerates. Bile-colored drainage or pus, redness, swelling, severe pain, and fever may all indicate infection. The bandage may be removed the day after surgery, and the patient can shower.

PT1. The nurse is caring for a woman recently diagnosed with viral hepatitis A. Which individual should the nurse refer for an immunoglobin (IG) injection? A. A caregiver who lives in the same household with the patient B. A friend who delivers meals to the patient and family each week C. A relative with a history of hepatitis A who visits the patient daily D. A child living in the home who received the hepatitis A vaccine 3 months ago

Ans: A IG is recommended for persons who do not have anti-HAV antibodies and are exposed as a result of close contact with persons who have HAV or foodborne exposure. Persons who have received a dose of HAV vaccine more than 1 month previously or who have a history of HAV infection do not require IG.

14. The patient with cirrhosis is being taught self-care. Which statement indicates the patient needs more teaching? A. "If I notice a fast heart rate or irregular beats, this is normal for cirrhosis." B. "I need to take good care of my belly and ankle skin where it is swollen." C. "A scrotal support may be more comfortable when I have scrotal edema." D. "I can use pillows to support my head to help me breathe when I am in bed."

Ans: A If the patient with cirrhosis experiences a fast or irregular heart rate, it may be indicative of hypokalemia and should be reported to the health care provider, as this is not normal for cirrhosis. Edematous tissue is subject to breakdown and needs meticulous skin care. Pillows and a semi-Fowler's or Fowler's position will increase respiratory efficiency. A scrotal support may improve comfort if there is scrotal edema.

6. A patient with type 2 diabetes and cirrhosis asks the nurse if it would be okay to take silymarin (milk thistle) to help minimize liver damage. The nurse responds based on what knowledge? A. Milk thistle may affect liver enzymes and thus alter drug metabolism. B. Milk thistle is generally safe in recommended doses for up to 10 years. C. There is unclear scientific evidence for the use of milk thistle in treating cirrhosis. D. Milk thistle may elevate the serum glucose levels and is thus contraindicated in diabetes.

Ans: A There is good scientific evidence that there is no real benefit from using milk thistle to protect the liver cells from toxic damage in the treatment of cirrhosis. Milk thistle does affect liver enzymes and thus could alter drug metabolism. Therefore patients will need to be monitored for drug interactions. It is noted to be safe for up to 6 years, not 10 years, and it may lower, not elevate, blood glucose levels

13. The patient with cirrhosis has an increased abdominal girth from ascites. The nurse should know that this fluid gathers in the abdomen for which reasons (select all that apply)? A. There is decreased colloid oncotic pressure from the liver's inability to synthesize albumin. Correct B. Hyperaldosteronism related to damaged hepatocytes increases sodium and fluid retention. Correct C. Portal hypertension pushes proteins from the blood vessels, causing leaking into the peritoneal cavity. Correct D. Osmoreceptors in the hypothalamus stimulate thirst, which causes the stimulation to take in fluids orally. E. Overactivity of the enlarged spleen results in increased removal of blood cells from the circulation, which decreases the vascular pressure.

Ans: A, B, C The ascites related to cirrhosis are caused by decreased colloid oncotic pressure from the lack of albumin from liver inability to synthesize it and the portal hypertension that shifts the protein from the blood vessels to the peritoneal cavity, and hyperaldosteronism which increases sodium and fluid retention. The intake of fluids orally and the removal of blood cells by the spleen do not directly contribute to ascites.

7. When caring for a patient with a biliary obstruction, the nurse will anticipate administering which vitamin supplements (select all that apply)? A. Vitamin A B. Vitamin D C. Vitamin E D. Vitamin K E. Vitamin B

Ans: A, B, C, D Biliary obstruction prevents bile from entering the small intestine and thus prevents the absorption of fat-soluble vitamins. Vitamins A, D, E, and K are all fat-soluble and thus would need to be supplemented in a patient with biliary obstruction

5. When caring for a patient with liver disease, the nurse recognizes the need to prevent bleeding resulting from altered clotting factors and rupture of varices. Which nursing interventions would be appropriate to achieve this outcome (select all that apply)? A. Use smallest gauge needle possible when giving injections or drawing blood. B. Teach patient to avoid straining at stool, vigorous blowing of nose, and coughing. C. Advise patient to use soft-bristle toothbrush and avoid ingestion of irritating food. D. Apply gentle pressure for the shortest possible time period after performing venipuncture. E. Instruct patient to avoid aspirin and NSAIDs to prevent hemorrhage when varices are present.

Ans: A, B, C, E Using the smallest gauge needle for injections will minimize the risk of bleeding into the tissues. Avoiding straining, nose blowing, and coughing will reduce the risk of hemorrhage at these sites. The use of a soft-bristle toothbrush and avoidance of irritating food will reduce injury to highly vascular mucous membranes. The nurse should apply gentle but prolonged pressure to venipuncture sites to minimize the risk of bleeding. Aspirin and NSAIDs should not be used in patients with liver disease because they interfere with platelet aggregation, thus increasing the risk for bleeding.

PT3. The nurse is caring for a 55-year-old man patient with acute pancreatitis resulting from gallstones. Which clinical manifestation would the nurse expect the patient to exhibit? A. Hematochezia B. Left upper abdominal pain C. Ascites and peripheral edema D. Temperature over 102o F (38.9o C)

Ans: B Abdominal pain (usually in the left upper quadrant) is the predominant manifestation of acute pancreatitis. Other manifestations of acute pancreatitis include nausea and vomiting, low-grade fever, leukocytosis, hypotension, tachycardia, and jaundice. Abdominal tenderness with muscle guarding is common. Bowel sounds may be decreased or absent. Ileus may occur and causes marked abdominal distention. Areas of cyanosis or greenish to yellow-brown discoloration of the abdominal wall may occur. Other areas of ecchymoses are the flanks (Grey Turner's spots or sign, a bluish flank discoloration) and the periumbilical area (Cullen's sign, a bluish periumbilical discoloration).

16. The patient with sudden pain in the left upper quadrant radiating to the back and vomiting was diagnosed with acute pancreatitis. What intervention(s) should the nurse expect to include in the patient's plan of care? A. Immediately start enteral feeding to prevent malnutrition. B. Insert an NG and maintain NPO status to allow pancreas to rest. C. Initiate early prophylactic antibiotic therapy to prevent infection. Incorrect D. Administer acetaminophen (Tylenol) every 4 hours for pain relief.

Ans: B Initial treatment with acute pancreatitis will include an NG tube if there is vomiting and being NPO to decrease pancreatic enzyme stimulation and allow the pancreas to rest and heal. Fluid will be administered to treat or prevent shock. The pain will be treated with IV morphine because of the NPO status. Enteral feedings will only be used for the patient with severe acute pancreatitis in whom oral intake is not resumed. Antibiotic therapy is only needed with acute necrotizing pancreatitis and signs of infection.

10. The patient with right upper quadrant abdominal pain has an abdominal ultrasound that reveals cholelithiasis. What should the nurse expect to do for this patient? A. Prevent all oral intake. B. Control abdominal pain. C. Provide enteral feedings. D. Avoid dietary cholesterol.

Ans: B Patients with cholelithiasis can have severe pain, so controlling pain is important until the problem can be treated. NPO status may be needed if the patient will have surgery but will not be used for all patients with cholelithiasis. Enteral feedings should not be needed, and avoiding dietary cholesterol is not used to treat cholelithiasis.

PT2. The nurse provides discharge instructions for a 64-year-old woman with ascites and peripheral edema related to cirrhosis. Which statement, if made by the patient, indicates teaching was effective? A. "It is safe to take acetaminophen up to four times a day for pain." B. "Lactulose (Cephulac) should be taken every day to prevent constipation." C. "Herbs and other spices should be used to season my foods instead of salt." D. "I will eat foods high in potassium while taking spironolactone (Aldactone)."

Ans: C A low-sodium diet is indicated for the patient with ascites and edema related to cirrhosis. Table salt is a well-known source of sodium and should be avoided. Alternatives to salt to season foods include the use of seasonings such as garlic, parsley, onion, lemon juice, and spices. Pain medications such as acetaminophen, aspirin, and ibuprofen should be avoided as these medications may be toxic to the liver. The patient should avoid potentially hepatotoxic over-the-counter drugs (e.g., acetaminophen) because the diseased liver is unable to metabolize these drugs. Spironolactone is a potassium-sparing diuretic. Lactulose results in the acidification of feces in bowel and trapping of ammonia, causing its elimination in feces

A 61-year-old patient with suspected bowel obstruction had a nasogastric tube inserted at 4:00 AM. The nurse shares in the morning report that the day shift staff should check the tube for patency at what times? A) 7:00 AM, 10:00 AM, and 1:00 PM B) 8:00 AM, 12:00 PM, and 4:00 PM C) 9:00 AM and 3:00 PM D) 9:00 AM, 12:00 PM, and 3:00 PM

B) A nasogastric tube should be checked for patency routinely at 4-hour intervals. Thus if the tube were inserted at 4:00 AM, it would be due to be checked at 8:00 AM, 12:00 PM, and 4:00 PM.

11. A patient with cholelithiasis needs to have the gallbladder removed. Which patient assessment is a contraindication for a cholecystectomy? A. Low-grade fever of 100° F and dehydration B. Abscess in the right upper quadrant of the abdomen C. Activated partial thromboplastin time (aPTT) of 54 seconds. D. Multiple obstructions in the cystic and common bile duct

Ans: C An aPTT of 54 seconds is above normal and indicates insufficient clotting ability. If the patient had surgery, significant bleeding complications postoperatively are very likely. Fluids can be given to eliminate the dehydration; the abscess can be assessed, and the obstructions in the cystic and common bile duct would be relieved with the cholecystectomy

PT5. The nurse instructs a 50-year-old woman about cholestyramine to reduce pruritis caused by gallbladder disease. Which statement by the patient to the nurse indicates she understands the instructions? A. "This medication will help me digest fats and fat-soluble vitamins." B. "I will apply the medicated lotion sparingly to the areas where I itch." C. "The medication is a powder and needs to be mixed with milk or juice." Correct D. "I should take this medication on an empty stomach at the same time each day."

Ans: C For treatment of pruritus, cholestyramine may provide relief. This is a resin that binds bile salts in the intestine, increasing their excretion in the feces. Cholestyramine is in powder form and should be mixed with milk or juice before oral administration.

9. The condition of a patient who has cirrhosis of the liver has deteriorated. Which diagnostic study would help determine if the patient has developed liver cancer? A. Serum α-fetoprotein level B. Ventilation/perfusion scan C. Hepatic structure ultrasound D. Abdominal girth measurement

Ans: C Hepatic structure ultrasound, CT, and MRI are used to screen and diagnose liver cancer. Serum α-fetoprotein level may be elevated with liver cancer or other liver problems. Ventilation/perfusion scans do not diagnose liver cancer. Abdominal girth measurement would not differentiate between cirrhosis and liver cancer

PT4. The nurse is caring for a group of patients. Which patient is at highest risk for pancreatic cancer? A. A 38-year-old Hispanic female who is obese and has hyperinsulinemia B. A 23-year-old who has cystic fibrosis-related pancreatic enzyme insufficiency C. A 72-year-old African American male who has smoked cigarettes for 50 years Correct D. A 19-year-old who has a 5-year history of uncontrolled type 1 diabetes mellitus

Ans: C Risk factors for pancreatic cancer include chronic pancreatitis, diabetes mellitus, age, cigarette smoking, family history of pancreatic cancer, high-fat diet, and exposure to chemicals such as benzidine. African Americans have a higher incidence of pancreatic cancer than whites. The most firmly established environmental risk factor is cigarette smoking. Smokers are two or three times more likely to develop pancreatic cancer as compared with nonsmokers. The risk is related to duration and number of cigarettes smoked.

1. A 54-year-old patient admitted with diabetes mellitus, malnutrition, osteomyelitis, and alcohol abuse has a serum amylase level of 280 U/L and a serum lipase level of 310 U/L. To what diagnosis does the nurse attribute these findings? a.Malnutrition b.Osteomyelitis c.Alcohol abuse d.Diabetes mellitus

Ans: C The patient with alcohol abuse could develop pancreatitis as a complication, which would increase the serum amylase (normal 30-122 U/L) and serum lipase (normal 31-186 U/L) levels as shown.

4. When planning care for a patient with cirrhosis, the nurse will give highest priority to which nursing diagnosis? A. Impaired skin integrity related to edema, ascites, and pruritus B. Imbalanced nutrition: less than body requirements related to anorexia C. Excess fluid volume related to portal hypertension and hyperaldosteronism D. Ineffective breathing pattern related to pressure on diaphragm and reduced lung volume

Ans: D Although all of these nursing diagnoses are appropriate and important in the care of a patient with cirrhosis, airway and breathing are always the highest priorities.

15. The patient with a history of lung cancer and hepatitis C has developed liver failure and is considering liver transplantation. After the comprehensive evaluation, the nurse knows that which factor discovered may be a contraindication for liver transplantation? A. Has completed a college education B. Has been able to stop smoking cigarettes Incorrect C. Has well-controlled type 1 diabetes mellitus D. The chest x-ray showed another lung cancer lesion.

Ans: D Contraindications for liver transplant include severe extrahepatic disease, advanced hepatocellular carcinoma or other cancer, ongoing drug and/or alcohol abuse, and the inability to comprehend or comply with the rigorous post-transplant course.

2. The health care provider orders lactulose for a patient with hepatic encephalopathy. The nurse will monitor for effectiveness of this medication for this patient by assessing what? A. Relief of constipation B. Relief of abdominal pain C. Decreased liver enzymes D. Decreased ammonia levels

Ans: D Hepatic encephalopathy is a complication of liver disease and is associated with elevated serum ammonia levels. Lactulose traps ammonia in the intestinal tract. Its laxative effect then expels the ammonia from the colon, resulting in decreased serum ammonia levels and correction of hepatic encephalopathy.

3. The family of a patient newly diagnosed with hepatitis A asks the nurse what they can do to prevent becoming ill themselves. Which response by the nurse is mostappropriate? A. "The hepatitis vaccine will provide immunity from this exposure and future exposures." B. "I am afraid there is nothing you can do since the patient was infectious before admission." C. "You will need to be tested first to make sure you don't have the virus before we can treat you." D. "An injection of immunoglobulin will need to be given to prevent or minimize the effects from this exposure."

Ans: D Immunoglobulin provides temporary (1-2 months) passive immunity and is effective for preventing hepatitis A if given within 2 weeks after exposure. It may not prevent infection in all persons, but it will at least modify the illness to a subclinical infection. The hepatitis vaccine is only used for preexposure prophylaxis

A patient who has hepatitis B surface antigen (HBsAg) in the serum is being discharged with pain medication after knee surgery. Which medication order should the nurse question because it is most likely to cause hepatic complications? A. Tramadol (Ultram) B. Hydromorphone (Dilaudid) C. Oxycodone with aspirin (Percodan) D. Hydrocodone with acetaminophen (Vicodin)

Ans: D The analgesic with acetaminophen should be questioned because this patient is a chronic carrier of hepatitis B and is likely to have impaired liver function. Acetaminophen is not suitable for this patient because it is converted to a toxic metabolite in the liver after absorption, increasing the risk of hepatocellular damage.

17. The patient with suspected pancreatic cancer is having many diagnostic studies done. Which one can be used to establish the diagnosis of pancreatic adenocarcinoma and for monitoring the response to treatment? A. Spiral CT scan Incorrect B. A PET/CT scan C. Abdominal ultrasound D. Cancer-associated antigen 19-9

Ans: D The cancer-associated antigen 19-9 (CA 19-9) is the tumor marker used for the diagnosis of pancreatic adenocarcinoma and for monitoring the response to treatment. Although a spiral CT scan may be the initial study done and provides information on metastasis and vascular involvement, this test and the PET/CT scan or abdominal ultrasound do not provide additional information.

12. When teaching the patient with acute hepatitis C (HCV), the patient demonstrates understanding when the patient makes which statement? A. "I will use care when kissing my wife to prevent giving it to her." B. "I will need to take adofevir (Hepsera) to prevent chronic HCV." C. "Now that I have had HCV, I will have immunity and not get it again." D. "I will need to be checked for chronic HCV and other liver problems."

Ans: D The majority of patients who acquire HCV usually develop chronic infection, which may lead to cirrhosis or liver cancer. HCV is not transmitted via saliva, but percutaneously and via high-risk sexual activity exposure. The treatment for acute viral hepatitis focuses on resting the body and adequate nutrition for liver regeneration. Adofevir (Hepsera) is taken for severe hepatitis B (HBV) with liver failure. Chronic HCV is treated with pegylated interferon with ribavirin. Immunity with HCV does not occur as it does with HAV and HBV, so the patient may be reinfected with another type of HCV.

Describe what is indicated by the ABCDEs of malignant melanoma. A B C D E

Asymmetry: one half unlike the other half; Border: irregular and poorly circumscribed; Color: varied within lesion; Diameter: larger than 6 mm; Evolving: look and appearance is changing

Which area of the abdomen will the nurse palpate to assess for splenomegaly? a. Right Upper Quadrant b. Left Upper Quadrant c. Right Lower Quadrant d. Left Lower Quadrant

B

The results of a patients recent endoscopy indicate the presence of peptic ulcer disease (PUD). Which of the following teaching points should the nurse provide to the pt in light of his new diagnosis? A) "You'll need to drink at least two to three glasses of milk daily." B) "It would likely be beneficial for you to eliminate drinking alcohol." C) "Many people find that a minced or pureed diet eases their sxs of PUD." D) "Your medications should allow you to maintain your present diet while minimizing symptoms."

B) "It would likely be beneficial for you to eliminate drinking alcohol." Although there is no specific recommended dietary modification for PUD, most patients find it necessary to make some sort of dietary modifications to minimize symptoms. Milk may exacerbate PUD and alcohol is best avoided because it can delay healing.

The nurse is planning care for a 68-year-old patient with an abdominal mass and suspected bowel obstruction. Which factor in the patient's history increases the patient's risk for colorectal cancer? A) Osteoarthritis B) History of colorectal polyps C) History of lactose intolerance D) Use of herbs as dietary supplements

B) A history of colorectal polyps places this patient at risk for colorectal cancer. This tissue can degenerate over time and become malignant. Osteoarthritis, lactose intolerance, and the use of herbs do not pose additional risk to the patient.

A patient is seeking emergency care after choking on a piece of steak. The nursing assessment reveals a history of alcoholism, cigarette smoking, and hemoptysis. Which diagnostic study is most likely to be performed on this patient? A) Barium swallow B) Endoscopic biopsy C) Capsule endoscopy D) Endoscopic ultrasonography

B) Because of this patient's history of excessive alcohol intake, smoking, hemoptysis, and the current choking episode, cancer may be present. A biopsy is necessary to make a definitive diagnosis of carcinoma, so an endoscope will be used to obtain a biopsy and observe other abnormalities as well. A barium swallow may show narrowing of the esophagus, but it is more diagnostic for achalasia. An endoscopic ultrasonography may be used to stage esophageal cancer. Capsule endoscopy can show alterations in the esophagus but is more often used for small intestine problems. A barium swallow, capsule endoscopy, and endoscopic ultrasonography cannot provide a definitive diagnosis for cancer when it is suspected.

A patient who is given a bisacodyl (Dulcolax) suppository asks the nurse how long it will take to work. The nurse replies that the patient will probably need to use the bedpan or commode within which time frame after administration? A) 2-5 minutes B) 15-60 minutes C) 2-4 hours D) 6-8 hours

B) Bisacodyl suppositories usually are effective within 15 to 60 minutes of administration, so the nurse should plan accordingly to assist the patient to use the bedpan or commode.

The nurse is caring for a patient treated with intravenous fluid therapy for severe vomiting. As the pt recovers and begins to tolerate oral intake, the N understands that which of the following food choices would be most appropriate? A) Ice tea B) Dry toast C) Warm broth D) Plain hamburger

B) Dry toast (Dry toast or crackers may alleviate the feeling of nausea and prevent further vomiting. Extremely hot or cold liquids and fatty foods are generally not well tolerated.)

The pt who is admitted with a diagnosis of diverticulitis and a history of irritable bowel disease and gastroesophageal reflux disease (GERD) has received a dose of Mylanta 30 ml PO. The nurse would evaluate its effectiveness by questioning the patient as to whether which of the following sxs has been resolved? A) Diarrhea B) Heartburn C) Constipation D) Lower abdominal pain

B) Heartburn (Mylanta is an antacid that contains both aluminum and magnesium. It is indicated for the relief of GI discomfort, such as with heartburn associated with GERD.)

The patient with sudden pain in the left upper quadrant radiating to the back and vomiting was diagnosed with acute pancreatitis. What intervention(s) should the nurse expect to include in the patient's plan of care? A) Immediately start enteral feeding to prevent malnutrition. B) Insert an NG and maintain NPO status to allow pancreas to rest. C) Initiate early prophylactic antibiotic therapy to prevent infection. D) Administer acetaminophen (Tylenol) every 4 hours for pain relief.

B) Initial treatment with acute pancreatitis will include an NG tube if there is vomiting and being NPO to decrease pancreatic enzyme stimulation and allow the pancreas to rest and heal. Fluid will be administered to treat or prevent shock. The pain will be treated with IV morphine because of the NPO status. Enteral feedings will only be used for the patient with severe acute pancreatitis in whom oral intake is not resumed. Antibiotic therapy is only needed with acute necrotizing pancreatitis and signs of infection.

The nurse should administer a prn dose of magnesium hydroxide (MOM) after noting which of the following while reviewing a patient's medical record? A) Abdominal pain and bloating B) No bowel movement for 3 days C) A decrease in appetite by 50% over 24 hours D) Muscle tremors and other signs of hypomagnesemia

B) MOM is an osmotic laxative that produces a soft, semisolid stool usually within 15 minutes to 3 hours. This medication would benefit the patient who has not had a bowel movement for 3 days. MOM would not be given for abdominal pain and bloating, decreased appetite, or signs of hypomagnesemia.

The patient with right upper quadrant abdominal pain has an abdominal ultrasound that reveals cholelithiasis. What should the nurse expect to do for this patient? A) Prevent all oral intake. B) Control abdominal pain. C) Provide enteral feedings. D) Avoid dietary cholesterol.

B) Patients with cholelithiasis can have severe pain, so controlling pain is important until the problem can be treated. NPO status may be needed if the patient will have surgery but will not be used for all patients with cholelithiasis. Enteral feedings should not be needed, and avoiding dietary cholesterol is not used to treat cholelithiasis.

A colectomy is scheduled for a 38-year-old woman with ulcerative colitis. The nurse should plan to include what prescribed measure in the preoperative preparation of this patient? A) Instruction on irrigating a colostomy B) Administration of a cleansing enema C) A high-fiber diet the day before surgery D) Administration of IV antibiotics for bowel preparation

B) Preoperative preparation for bowel surgery typically includes bowel cleansing with antibiotics, such as oral neomycin and cleansing enemas, including Fleet enemas. Instructions to irrigate the colostomy will be done postoperatively. Oral antibiotics are given preoperatively, and an IV antibiotic may be used in the OR. A clear liquid diet will be used the day before surgery with the bowel cleansing.

The patient had a car accident and was "scared to death." The patient is now reporting constipation. What affecting the gastrointestinal (GI) tract does the nurse know could be contributing to the constipation? A) The patient is too nervous to eat or drink, so there is no stool. B) The sympathetic nervous system was activated, so the GI tract was slowed. C) The parasympathetic nervous system is now functioning to slow the GI tract. D) The circulation in the GI system has been increased, so less waste is removed.

B) The constipation is most likely related to the sympathetic nervous system activation from the stress related to the accident. SNS activation can decrease peristalsis. Even without oral intake for a short time, stool will be formed. The parasympathetic system stimulates peristalsis. The circulation to the GI system is decreased with stress.

Inspection of an older patient's mouth reveals the presence of white, curd-like lesions on the patient's tongue. What is the most likely etiology for this abnormal assessment finding? A) Herpesvirus B) Candida albicans C) Vitamin deficiency D) Irritation from ill-fitting dentures

B) White, curd-like lesions surrounded by erythematous mucosa are associated with oral candidiasis. Herpesvirus causes benign vesicular lesions in the mouth. Vitamin deficiencies may cause a reddened, ulcerated, swollen tongue. Irritation from ill-fitting dentures will cause friable, edematous, painful, bleeding gingivae.

The nurse should recognize that the liver performs which functions (select all that apply)? A) Bile storage B) Detoxification C) Protein metabolism D) Steroid metabolism E) Red blood cell (RBC) destruction

B, C, D) The liver performs multiple major functions that aid in the maintenance of homeostasis. These include metabolism of proteins and steroids as well as detoxification of drugs and metabolic waste products. The Kupffer cells of the liver participate in the breakdown of old RBCs. The liver produces bile, but storage occurs in the gall bladder.

A 19-year-old patient reports to the clinic nurse the following symptoms: a ring-like itchy rash on the upper leg, low-grade fever, nausea, and joint pain for the past 3 weeks. What question is important for the nurse to ask the patient? A. "Is the itching worse at night?" B. "Have you had a tick bite recently?" C. "Have you been exposed to pubic lice?" D. "Have you had unprotected sexual contact?"

B. "Have you had a tick bite recently?" Symptoms are consistent with Lyme disease caused by the organism Borrelia burgdorferi, which is transmitted by a tick bite.

1. A client asks a nurse why the provider bases his medication regimen on his HbA1c instead of his log of morning fasting blood glucose results. Which of the following is an appropriate response by the nurse? A. "HbA1c measures how well insulin is regulating your blood glucose between meals." B. "HbA1c indicates how well your blood glucose has been regulated over the past 3 months." HbA1c measures the client's blood glucose control over the past 2 to 3 months. C. "A test of HbA1c is the first test to determine if an individual has diabetes." D. "A test of HbA1c determines if the dosage of insulin needs to be adjusted."

B. "HbA1c indicates how well your blood glucose has been regulated over the past 3 months." HbA1c measures the client's blood glucose control over the past 2 to 3 months.

The nurse teaches a 50-year-old woman with chronic kidney disease several interventions to reduce pruritus associated with dry skin and uremia. Which statement, if made by the patient to the nurse, indicates further teaching is required? A. "I will avoid taking hot showers." B. "I can rub my skin instead of scratching." C. "Menthol can be used to numb the itch sensation." D. "A lubricating lotion right after bathing will help."

B. "I can rub my skin instead of scratching." Any activity that causes vasodilation, such as rubbing or bathing and showering in hot water should be avoided as vasodilation leads to increased itching. Menthol in skin products provides a sensation that may distract the patient from the sensation of itchiness. Applying lotion right after bathing helps retain moisture in the skin.

The nurse should recognize which patient as likely to have the poorest prognosis? A. A 60-year-old diagnosed with nodular ulcerative basal cell carcinoma B. A 59-year-old man who is being treated for stage IV malignant melanoma C. A 70-year-old woman who has been diagnosed with late squamous cell carcinoma D. A 51-year-old woman whose biopsy has revealed superficial squamous cell carcinoma

B. A 59-year-old man who is being treated for stage IV malignant melanoma Late detection of malignant melanoma is associated with a poor outcome. Basal cell carcinomas often have very effective treatment success rates. Although late squamous cell carcinoma (SCC) has worse outcomes than superficial SCC, these are both exceeded in mortality by late-stage malignant melanoma.

2. A nurse is preparing to administer a morning dose of aspart insulin (NovoLog) to a client who has type 1 diabetes mellitus. Which of the following is an appropriate action by the nurse? A. Check the client's blood glucose immediately after breakfast. B. Administer the insulin when breakfast arrives. C. Hold breakfast for 1 hr after insulin administration. D. Clarify the prescription because insulin should not be administered at this time.

B. Administer the insulin when breakfast arrives.

2. A nurse is caring for a client who has diabetes insipidus. Which of the following urinalysis laboratory findings should the nurse anticipate? A. Absence of glucose B. Decreased specific gravity C. Presence of ketones D. Presence of red blood cells

B. Decreased specific gravity

1. A nurse in a provider's office is reviewing the health record of a client who is being evaluated for Graves' disease. Which of the following is an expected laboratory finding for this client? A. Decreased thyrotropin receptor antibodies B. Decreased thyroid stimulating hormone C. Decreased free thyroxine index D. Decreased triiodothyronine

B. Decreased thyroid stimulating hormone

2. A nurse is assessing a client who has diabetic ketoacidosis and ketones in the urine. Which of the following are expected findings? (Select all that apply.) A. Weight gain B. Fruity odor of breath C. Abdominal pain D. Kussmaul respirations E. Metabolic acidosis

B. Fruity odor of breath C. Abdominal pain D. Kussmaul respirations E. Metabolic acidosis

2. A nurse is reviewing the clinical manifestations of hyperthyroidism with a client. Which of the following findings should the nurse include? (Select all that apply.) A. Dry skin B. Heat intolerance C. Constipation D. Palpitations E. Weight loss F. Bradycardia

B. Heat intolerance D. Palpitations E. Weight loss

3. A nurse is reinforcing teaching with a client who has been prescribed levothyroxine (Synthroid) to treat hypothyroidism. Which of the following should the nurse include in the teaching? (Select all that apply.) A. Weight gain is expected while taking this medication. B. Medication should not be discontinued without the advice of the provider. C. Follow-up serum TSH levels should be obtained. D. Take the medication on an empty stomach. E. Use fiber laxatives for constipation.

B. Medication should not be discontinued without the advice of the provider. C. Follow-up serum TSH levels should be obtained. D. Take the medication on an empty stomach.

2. A nurse is collecting an admission history from a female client who has hypothyroidism. Which of the following findings are expected with this condition? (Select all that apply.) A. Diarrhea B. Menorrhagia C. Dry skin D. Increased libido E. Hoarseness

B. Menorrhagia C. Dry skin E. Hoarseness

A nurse on the surgical unit is preparing to care for a client 12 hr post-total thyroidectomy. Which of the following interventions should the nurse anticipate implementing? (select all that apply) A. Restrict the client from speaking B. Monitor vital sign every 4 hour C. Keep the client in high-fowler's position D. Administer mild analgesics as prescribed E. Support the neck while client coughs and deep breathes every 2 hour

B. Monitor vital sign every 4 hour C. Keep the client in high-fowler's position D. Administer mild analgesics as prescribed E. Support the neck while client coughs and deep breathes every 2 hour

A nurse caring for a client recognizes that the client TSH is reliable indicator of the efficacy of the levothyroxine sodium because the TSH will A. Have a value of zero when when an euthyroid state is re-established B. Return to its expected reference range when an euthyroid state is reestablished C. Increase above its expected reference range when a therapeutic medication level is reached D. Decrease below it's expected reference range when a therapeutic medication level is reached

B. Return to its expected reference range when an euthyroid state is reestablished

3. A nurse is reviewing laboratory reports of a client who has hyperglycemic-hyperosmolar state (HHS). Which of the following is an expected finding? A. Serum pH 7.2 B. Serum osmolarity 350 mOsm/L C. Serum potassium 3.8 mg/dL D. Serum creatinine 0.8 mg/dL

B. Serum osmolarity 350 mOsm/L

3. A nurse is providing instructions to a client who has Graves' disease and has a new prescription for propranolol (Inderal). Which of the following information should the nurse include? A. An adverse effect of this medication is jaundice. B. Take your pulse before each dose. C. The purpose of this medication is to decrease production of thyroid hormone. D. You should stop taking this medication if you have a sore throat.

B. Take your pulse before each dose.

Which assessment finding of a 70-year-old male patient's skin should the nurse prioritize? A. The patient's complaint of dry skin that is frequently itchy B. The presence of an irregularly shaped mole that the patient states is new C. The presence of veins on the back of the patient's leg that are blue and tortuous D. The presence of a rash on the patient's hand and forearm to which the patient applies a corticosteroid ointment

B. The presence of an irregularly shaped mole that the patient states is new Although all of the noted assessment findings are significant, the presence of an irregular mole that is new is suggestive of a neoplasm and warrants immediate follow-up.

The patient has had rashes and alopecia. What vitamin in which foods should be encouraged as a nutritional aid to these problems? A. Vitamin A in sweet potatoes, carrots, dark leafy greens B. Vitamin B7 (biotin) in liver, cauliflower, salmon, carrots C. Vitamin C in peppers, dark leafy greens, broccoli, and kiwi D. Vitamin D in canned salmon, sardines, fortified dairy, and eggs

B. Vitamin B7 (biotin) in liver, cauliflower, salmon, carrots A deficiency of Vitamin B7 (biotin) may result in rashes and alopecia. Eating foods with biotin will help decrease these problems. Vitamins A and C are needed for wound healing. Vitamin D is needed for bone and body health.

Which laboratory result is the best indicator that a patient with cellulitis is recovering from this infection? A. WBC of 2900/μL B. WBC of 8200/μL C. WBC of 12,700/μL D. WBC of 16,300/μL

B. WBC of 8200/μL The normal white blood cell count is generally 4000 to 11,000/μL. For this reason, the patient's level would be returning to normal if it was 8200/μL, indicating recovery from cellulitis. The 2900/µL is too low and indicates another problem is occurring. The 12,700/µL and 16,300/µL are evidence of continuing infection.

The nurse cares for a 41-year-old male patient admitted for uncontrolled seizures who is also diagnosed with impetigo on the face and neck. Which action is appropriate for the nurse to take? A. Put on a protective gown before entering the room. B. Wash hands for 1 to 2 minutes when leaving the room. C. Wear gloves to leave a diet menu on the patient's table. D. Wear a particulate mask when within 3 feet of the patient.

B. Wash hands for 1 to 2 minutes when leaving the room. Impetigo is a bacterial skin infection with group A β-hemolytic streptococci or staphylococci. Meticulous hygiene (including hand washing) is essential to prevent the spread of infection. A particulate mask or a gown would not be necessary to prevent the spread of impetigo. Gloves would not be needed to make a delivery to the room.

A patient who is scheduled for surgery with general anesthesia in 1 hour is observed with a moist, but empty water glass in his hand. Which assessment finding may indicate that the patient drank a glass of water? A) Flat abdomen without movement upon inspection B) Tenderness at left upper quadrant upon palpation C) Easily heard, loud gurgling in the right upper quadrant D) High-pitched, hollow sounds in the left upper quadrant

C) If the patient drank water on an empty stomach, gurgling can be assessed without a stethoscope or assessed with auscultation. High-pitched, hollow sounds are tympanic and indicate an empty cavity. A flat abdomen and tenderness do not indicate that the patient drank a glass of water.

The patient tells the nurse she had a history of abdominal pain, so she had a surgery to make an opening into the common bile duct to remove stones. The nurse knows that this surgery is called a A) colectomy B) cholecystectomy C) choledocholithotomy D) choledochojejunostomy

C) A choledocholithotomy is an opening into the common bile duct for the removal of stones. A colectomy is the removal of the colon. The cholecystectomy is the removal of the gallbladder. The choledochojejunostomy is an opening between the common bile duct and the jejunum.

A patient with cholelithiasis needs to have the gallbladder removed. Which patient assessment is a contraindication for a cholecystectomy? A) Low-grade fever of 100° F and dehydration B) Abscess in the right upper quadrant of the abdomen C) Activated partial thromboplastin time (aPTT) of 54 seconds D) Multiple obstructions in the cystic and common bile duct

C) An aPTT of 54 seconds is above normal and indicates insufficient clotting ability. If the patient had surgery, significant bleeding complications postoperatively are very likely. Fluids can be given to eliminate the dehydration; the abscess can be assessed, and the obstructions in the cystic and common bile duct would be relieved with the cholecystectomy.

When assessing a patient's abdomen, what would be most appropriate for the nurse to do? A) Palpate the abdomen before auscultation. B) Percuss the abdomen before auscultation. C) Auscultate the abdomen before palpation. D) Perform deep palpation before light palpation.

C) During examination of the abdomen, auscultation is done before percussion and palpation because these latter procedures may alter the bowel sounds.

A female patient has a sliding hiatal hernia. What nursing interventions will prevent the symptoms of heartburn and dyspepsia that she is experiencing? A) Keep the patient NPO. B) Put the bed in the Trendelenberg position. C) Have the patient eat 4 to 6 smaller meals each day. D) Give various antacids to determine which one works for the patient.

C) Eating smaller meals during the day will decrease the gastric pressure and the symptoms of hiatal hernia. Keeping the patient NPO or in a Trendelenberg position are not safe or realistic for a long period of time for any patient. Varying antacids will only be done with the care provider's prescription, so this is not a nursing intervention.

The health care team is assessing a male patient for acute pancreatitis after he presented to the emergency department with severe abdominal pain. Which laboratory value is the best diagnostic indicator of acute pancreatitis? A) Gastric pH B) Blood glucose C) Serum amylase D) Serum potassium

C) Elevated serum amylase levels indicate early pancreatic dysfunction and are used to diagnose acute pancreatitis. Serum lipase levels stay elevated longer than serum amylase in acute pancreatitis. Blood glucose, gastric pH, and potassium levels are not direct indicators of acute pancreatic dysfunction.

The nurse determines that a pnt has experienced the beneficial effects of medication therapy with famotidine (Pepcid) when which of the following symptoms is relieved? A) Nausea B) Belching C) Epigastric pain D) Difficulty swallowing

C) Epigastric pain Famotidine is an H2-receptor antagonist that inhibits parietal cell output of HCl acid and minimizes damage to gastric mucosa related to hyperacidity, thus relieving epigastric pain.

The condition of a patient who has cirrhosis of the liver has deteriorated. Which diagnostic study would help determine if the patient has developed liver cancer? A) Serum α-fetoprotein level B) Ventilation/perfusion scan C) Hepatic structure ultrasound D) Abdominal girth measurement

C) Hepatic structure ultrasound, CT, and MRI are used to screen and diagnose liver cancer. Serum α-fetoprotein level may be elevated with liver cancer or other liver problems. Ventilation/perfusion scans do not diagnose liver cancer. Abdominal girth measurement would not differentiate between cirrhosis and liver cancer

Following administration of a dose of metoclopramide (Reglan) to the patient, the nurse determines that the medication has been effective when what is noted? A) Decreased blood pressure B) Absence of muscle tremors C) Relief of nausea and vomiting D) No further episodes of diarrhea

C) Metoclopramide is classified as a prokinetic and antiemetic medication. If it is effective, the patient's nausea and vomiting should resolve. Metoclopramide does not affect blood pressure, muscle tremors, or diarrhea.

The patient receiving chemotherapy rings the call bell and reports an onset of nausea. The nurse should prepare a prn dose of which of the following medications? A) Morphine sulfate B) Zolpidem (Ambien) C) Ondansetron (Zofran) D) Dexamethasone (Decadron)

C) Ondansetron (Zofran) (Ondansetron is a 5-HT3 receptor antagonist antiemetic that is especially effective in reducing cancer chemotherapy-induced nausea and vomiting.)

When caring for the patient with heart failure, the nurse knows that which gastrointestinal process is most dependent on cardiac output and may affect the patient's nutritional status? A) Ingestion B) Digestion C) Absorption D) Elimination

C) Substances that interface with the absorptive surfaces of the GI tract (primarily in the small intestine) diffuse across the intestinal membranes into intestinal capillaries and are then carried to other parts of the body for use in energy production. The cardiac output provides the blood flow for this absorption of nutrients to occur.

The patient is having an esophagoenterostomy with anastomosis of a segment of the colon to replace the resected portion. What initial postoperative care should the nurse expect when this patient returns to the nursing unit? A) Turn, deep breathe, cough, and use spirometer every 4 hours. B) Maintain an upright position for at least 2 hours after eating. C) NG will have bloody drainage, and it should not be repositioned. D) Keep in a supine position to prevent movement of the anastomosis.

C) The patient will have bloody drainage from the NG tube for 8 to 12 hours, and it should not be repositioned or reinserted without contacting the surgeon. Turning and deep breathing will be done every 2 hours, and the spirometer will be used more often than every 4 hours. Coughing would put too much pressure in the area and should not be done. Because the patient will have the NG tube, the patient will not be eating yet. The patient should be kept in a semi-Fowler's or Fowler's position, not supine, to prevent reflux and aspiration of secretions.

A 54-year-old patient admitted with diabetes mellitus, malnutrition, osteomyelitis, and alcohol abuse has a serum amylase level of 280 U/L and a serum lipase level of 310 U/L. To which of the following diagnoses does the nurse attribute these findings? A. Malnutrition B. Osteomyelitis C. Alcohol abuse D. Diabetes mellitus

C) The patient with alcohol abuse could develop pancreatitis as a complication, which would increase the serum amylase (normal 30-122 U/L) and serum lipase (normal 31-186 U/L) levels as shown.

Following bowel resection, a patient has a nasogastric (NG) tube to suction, but complains of nausea and abdominal distention. The nurse irrigates the tube as necessary as ordered, but the irrigating fluid does not return. What should be the priority action by the nurse? A) Notify the physician. B) Auscultate for bowel sounds. C) Reposition the tube and check for placement. D) Remove the tube and replace it with a new one.

C) The tube may be resting against the stomach wall. The first action by the nurse (since this is intestinal surgery and not gastric surgery) is to reposition the tube and check it again for placement. The physician does not need to be notified unless the tube function cannot be restored by the nurse. The patient does not have bowel sounds, which is why the NG tube is in place. The NG tube would not be removed and replaced unless it was no longer in the stomach or the obstruction of the tube could not be relieved.

The patient with chronic gastritis is being put on a combination of medications to eradicate H. pylori. Which drugs does the nurse know will probably be used for this patient? A) Antibiotic(s), antacid, and corticosteroid B) Antibiotic(s), aspirin, and antiulcer/protectant C) Antibiotic(s), proton pump inhibitor, and bismuth D) Antibiotic(s) and nonsteroidal antiinflammatory drugs (NSAIDs)

C) To eradicate H. pylori, a combination of antibiotics, a proton pump inhibitor, and possibly bismuth (for quadruple therapy) will be used. Corticosteroids, aspirin, and NSAIDs are drugs that can cause gastritis and do not affect H. pylori.

Which clinical manifestations of inflammatory bowel disease are common to both patients with ulcerative colitis (UC) and Crohn's disease (select all that apply)? A) Restricted to rectum B) Strictures are common. C) Bloody, diarrhea stools D) Cramping abdominal pain E) Lesions penetrate intestine.

C, D) Clinical manifestations of UC and Crohn's disease include bloody diarrhea, cramping abdominal pain, and nutritional disorders. Intestinal lesions associated with UC are usually restricted to the rectum before moving into the colon. Lesions that penetrate the intestine or cause strictures are characteristic of Crohn's disease.

The nurse is providing preoperative teaching for the patient having a face-lift (rhytidectomy) surgery. Which patient response indicates the patient understands the teaching? A. "I am afraid of the pain afterwards, while it is healing." B. "I can't wait to have my forehead and lip wrinkles eliminated." C. "I have some time off work so I will not look so bad when I go back." D. "Now I can be excited to go to my 50th high school reunion this week.

C. "I have some time off work so I will not look so bad when I go back." A rhytidectomy or face-lift surgery will not have immediate results and will take time to heal, so taking time off from work will allow more healing to be accomplished before returning to work. There is not much pain with most cosmetic surgeries. A rhytidectomy will not eliminate forehead lines and vertical lip wrinkles.

The nurse is teaching about skin cancer prevention at the community center. Which individual is most at risk for developing skin cancer? A. A 67-year-old bald-headed man with psoriasis and type 2 diabetes mellitus B. A 76-year-old Hispanic man who has a latex allergy and numerous acrochordons C. A 55-year-old woman with fair skin and red hair who has a family history of skin cancer D. A 62-year-old woman with chronic kidney disease who has blond hair with dry, pale skin and pruritus

C. A 55-year-old woman with fair skin and red hair who has a family history of skin cancer Risk factors for skin cancer include having fair skin (with red hair) and a family history of skin cancer. Allergies, acrochordons (skin tags), psoriasis, type 2 diabetes mellitus, and chronic kidney disease are not risk factors associated with the development of skin cancer.

3. A nurse is preparing to administer the morning doses of glargine (Lantus) insulin and regular (Humulin R) insulin to a client who has a blood glucose of 278 mg/dL. Which of the following is an appropriate nursing action? A. Draw up the regular insulin and then the glargine insulin in the same syringe. B. Draw up the glargine insulin then the regular insulin in the same syringe. C. Draw up and administer regular and glargine insulin in separate syringes. D. Administer the regular insulin, wait 1 hr, and then administer the glargine insulin.

C. Draw up and administer regular and glargine insulin in separate syringes.

1. A nurse in a provider's office is reviewing the laboratory findings of a client who is being evaluated for primary hypothyroidism. Which of the following laboratory findings is expected for a client who has this condition? A. Serum T4 10 mcg/dL B. Serum T3 200 ng/dL C. Hematocrit 34% D. Serum cholesterol 180 mg/dL

C. Hematocrit 34%

The patient with diabetes mellitus has peripheral vascular disease. Knowing this, for which dermatologic manifestations should the nurse expect to assess? A. Redness of exposed areas of the skin on the hand, foot, face, or neck and infected dermatitis B. Leathery, brownish skin on lower leg, pruritis, concave lesions with edema, scar tissue with healing C. Loss of hair in periphery, delayed capillary filling, dependent rubor, neuropathy, and delayed wound healing D. Atrophy, epidermal thinning, increased vascular fragility, impaired wound healing, thin loose dermis, and excess fat at the back of the neck

C. Loss of hair in periphery, delayed capillary filling, dependent rubor, neuropathy, and delayed wound healing The patient with diabetes mellitus and peripheral vascular disease is likely to have loss of peripheral hair, delayed capillary filling, dependent rubor, neuropathy, and delayed wound healing. The patient with a nicotinic acid (niacin) deficiency manifests redness of exposed areas of the skin on the hand or foot, face, or neck and infected dermatitis. The patient with venous ulcers will have leathery, brownish skin on the lower leg, pruritus, concave lesions with edema, and scar tissue with healing. The patient with glucocorticoid excess (Cushing syndrome) may have atrophy, epidermal thinning, increased vascular fragility, impaired wound healing, thin loose dermis, and excess fat at the back of the neck, clavicles, abdomen, and face.

5. A nurse is teaching foot care to a client who has diabetes mellitus. Which of the following information should the nurse include in the teaching? (Select all that apply.) A. Remove calluses using over-the-counter remedies. B. Apply lotion between toes. C. Perform nail care after bathing. D. Trim toenails straight across. E. Wear closed-toe shoes

C. Perform nail care after bathing. D. Trim toenails straight across. E. Wear closed-toe shoes

1. A nurse is caring for a client who has blood glucose of 52 mg/dL. The client is lethargic but arousable. Which of the following actions should the nurse perform first? A. Recheck blood glucose in 15 min. B. Provide a carbohydrate and protein food. C. Provide 4 oz grape juice. D. Report findings to the provider.

C. Provide 4 oz grape juice.

4. A nurse is preparing to administer IV fluids to a client who has diabetic ketoacidosis. Which of the following is an appropriate nursing action? A. Administer an IV infusion of regular insulin at 0.3 unit/kg/hr. B. Administer an IV infusion of 0.45% sodium chloride. C. Rapidly administer an IV infusion of 0.9% sodium chloride. D. Add glucose to the IV infusion when serum glucose is 350 mg/dL

C. Rapidly administer an IV infusion of 0.9% sodium chloride.

A 56-year-old white patient presents with a flat, dry, scaly area on her eyebrows that is treated with a chemical peel. What should the nurse include in the discharge teaching? A. Metastasis of this type of cancer is rare. B. The patient has an increased risk for melanoma. C. Recurrence of the premalignant lesion is possible. D. Untreated lesions may metastasize to regional lymph nodes.

C. Recurrence of the premalignant lesion is possible. The flat or elevated dry scaly area is actinic keratosis from sun damage and is a premalignant skin lesion common in older whites with possible recurrence even with adequate treatment. Metastasis of basal cell carcinoma is rare; it is a small slowly enlarging papule. There is an increased risk for melanoma with atypical or dysplastic nevi. With squamous cell carcinoma, untreated lesions may metastasize to regional lymph nodes and distant organs, but it has a high cure rate with early detection and treatment.

Which of the following should the nurse reinforce in regard to taking levothyroxine? (select all that apply) A. Use fiber laxative for constipation B. Expect to take this medication for 3-6 months C. Return to clinic for follow up lab tests of serum TSH D. Take medication 30 min before bedtime E. Side effect such as nervousness, heat intolerance, and diarrhea

C. Return to clinic for follow up lab tests of serum TSH E. Side effect such as nervousness, heat intolerance, and diarrhea

6. A nurse is assessing a client who is 12 hr postoperative following a thyroidectomy. Which of the following findings are indicative of thyroid crisis? (Select all that apply.) A. Bradycardia B. Hypothermia C. Tremors D. Abdominal pain E. Mental confusion

C. Tremors D. Abdominal pain E. Mental confusion

A nurse is caring for a client with hyperparathyroidism who has undergone surgical removal of the parathyroid glands. The knows that improvement in the client's condition is indicated by a decrease in serum?

Calcium

When preparing a patient for a capsule endoscopy study, what should the nurse do? a. Ensure the patient understands the required bowel preparation. b. Have the patient return to the procedure room for removal of the capsule. c. Teach the patient to maintain a clear liquid diet throughout the procedure. d. Explain to the patient that conscious sedation will be used during placement of the capsule.

Correct answer: a Rationale: A capsule endoscopy study involves the patient performing a bowel prep to cleanse the bowel before swallowing the capsule. The patient will be on a clear liquid diet for 1 to 2 days before the procedure and will remain NPO for 4 to 6 hours after swallowing the capsule. The capsule is disposable and will pass naturally with the bowel movement, although the monitoring device will need to be removed.

What problem should the nurse assess the patient for if the patient was on prolonged antibiotic therapy? a. Coagulation problems b. Elevated serum ammonia levels c. Impaired absorption of amino acids d. Increased mucus and bicarbonate secretion

Correct answer: a Rationale: Bacteria int he colon (1) synthesize vitamin K, which is needed for the production of prothrombin by the liver and (2) deaminate undigested or non absorbed proteins, producing ammonia, which is converted to urea by the liver. A reduction in normal flora bacteria by antibiotic therapy can lead to decreased vitamin K, resulting in decreased prothrombin and coagulation problems. Bowel bacteria do not influence protein absorption or the secretion of mucus.

What characterizes auscultation of the abdomen? a. The presence of borborygmi indicates hyper peristalsis. b. The bell of the stethoscope is used to auscultate high-pitched sounds. c. High-pitched, rushing, and tinkling bowel sounds are heard after eating. d. Absence of bowel sounds for 1 minute in each quadrant is reported as abnormal.

Correct answer: a Rationale: Borborygmi are loud gurgles (stomach growling) that indicate hyper peristalsis. Normal bowel sounds are relatively high-pitched and are heard best with the diaphragm of the stethoscope. High-pitched, tinkling bowel sounds occur when the intestines are under tension, as in bowel obstructions. Absent bowel sounds may be reported when no sounds are heard for 2 to 3 minutes in each quadrant.

A patient had a stomach resection for stomach cancer. The nurse should teach the patient about the loss of the hormone that stimulates gastric acid secretion and motility and maintains lower esophageal sphincter tone. Which hormone will be decreased with a gastric resection? a. Gastrin b. Secretin c. Cholecystokinin d. Gastric inhibitory peptide

Correct answer: a Rationale: Gastrin is the hormone activated in the stomach (and duodenal mucosa) by stomach distention that stimulates gastric acid secretion and motility and maintains lower esophageal sphincter tone. Secretin inhibits gastric motility and acid secretion and stimulates pancreatic bicarbonate secretion. Cholecystokinin allows increased flow of bile into the duodenum and release of pancreatic digestive enzymes. Gastric inhibitory peptide inhibits gastric acid secretion and motility.

Priority Decision: Following auscultation of the abdomen, what should the nurse's next action be? a. Lightly percuss over all four quadrants b. Have the patient empty his or her bladder c. Inspect perianal and anal areas for color, masses, rashes, and scars d. Perform deep palpation to delineate abdominal organs and masses

Correct answer: a Rationale: The abdomen should be assessed in the following sequence: inspection, auscultation, percussion, palpation. The patient should empty his or her bladder before assessment begins.

Checking for the return of the gag reflex and monitoring for LUQ pain, nausea and vomiting are necessary nursing actions after which diagnostic procedure? a. ERCP b. Colonoscopy c. Barium swallow d. Esophagogastroduodenoscopy (EGD)

Correct answer: a Rationale: The left upper quadrant (LUQ) pain and nausea and vomiting could occur from perforation. The return of gag reflex is essential to prevent aspiration after an ERCP. The gag reflex is also assessed with an EGD. These are not relevant assessments for the colonoscopy and barium swallow.

A 35-year-old man with a family history of adenomatous polyposis had a colonoscopy with removal of multiple polyps. Which signs and symptoms should the nurse teach the patient to report immediately? a. Fever and abdominal pain b. Flatulence and liquid stool c. Loudly audible bowel sounds d. Sleepiness and abdominal cramps

Correct answer: a Rationale: The patient should be taught to observe for signs of rectal bleeding and peritonitis. Fever, malaise, and abdominal pain and distention could indicate a perforated bowel with peritonitis.

A 90-year-old healthy man is suffering from dysphagia. The nurse explains what age-related change of the GI tract is the most likely cause of his difficulty? a. Xerostomia b. Esophageal cancer c. Decreased taste buds d. Thinner abdominal wall

Correct answer: a Rationale: Xerostomia, decreased volume of saliva, leads to dry oral mucosa and dysphagia. Esophageal cancer is not an age-related change. Decreased taste buds and a thinner abdominal wall do not contribute to difficulty swallowing.

The patient had a car accident and was "scared to death." The patient is now reporting constipation. What affecting the gastrointestinal (GI) tract does the nurse know could be contributing to the constipation? a. The patient is too nervous to eat or drink, so there is no stool. b. The sympathetic nervous system was activated, so the GI tract was slowed. c. The parasympathetic nervous system is now functioning to slow the GI tract. d. The circulation in the GI system has been increased, so less waste is removed.

Correct answer: b Rationale: The constipation is most likely related to the sympathetic nervous system activation from the stress related to the accident. SNS activation can decrease peristalsis. Even without oral intake for a short time, stool will be formed. The parasympathetic system stimulates peristalsis. The circulation to the GI system is decreased with stress.

A patient is admitted to the hospital with left upper quadrant (LUQ) pain. What may be a possible source of the pain? a. Liver b. Pancreas c. Appendix d. Gallbladder

Correct answer: b Rationale: The pancreas is located in the left upper quadrant, the liver is in the right upper quadrant, the appendix is in the right lower quadrant, and the gallbladder is in the right upper quadrant.

A 62-year-old woman patient is scheduled for a percutaneous transhepatic cholangiography to restore biliary drainage. The nurse discusses the patient's health history and is most concerned if the patient makes which statement? a."I am allergic to bee stings." b. "My tongue swells when I eat shrimp." c. "I have had epigastric pain for 2 months." d. "I have a pacemaker because my heart rate was slow."

Correct answer: b Rationale: The percutaneous transhepatic cholangiography procedure will include the use of radiopaque contrast medium. Patients allergic to shellfish and iodine are also allergic to contrast medium. Having a pacemaker will not affect the patient during this procedure. It would be expected that the patient would have some epigastric pain given the patient's condition.

What is a normal finding on physical examination of the abdomen? a. Auscultation of bruits b. Observation of visible pulsations c. Percussion of liver dullness in the left midclavicular line d. Palpation of the spleen 1 to 2 cm below the left costal margin

Correct answer: b Rationale: The pulsation of the aorta in the epigastric area is a normal finding. Bruits indicate that blood flow is abnormal, the liver is percussed in the right midclavicular line, and a normal spleen cannot be palpated.

Inspection of an older patient's mouth reveals the presence of white, curd-like lesions on the patient's tongue. What is the most likely etiology for this abnormal assessment finding? a. Herpesvirus b. Candida albicans c. Vitamin deficiency d. Irritation from ill-fitting dentures

Correct answer: b Rationale: White, curd-like lesions surrounded by erythematous mucosa are associated with oral candidiasis. Herpesvirus causes benign vesicular lesions in the mouth. Vitamin deficiencies may cause a reddened, ulcerated, swollen tongue. Irritation from ill-fitting dentures will cause friable, edematous, painful, bleeding gingivae.

An 85-year-old woman seen in the primary care provider's office for a well check complains of difficulty swallowing. What common effect of aging should the nurse assess for as a possible cause? a. Anosmia b. Xerostomia c. Hypochlorhydria d. Salivary gland tumor

Correct answer: b Rationale: Xerostomia (decreased saliva production), or dry mouth, affects many older adults and may be associated with difficulty swallowing (dysphagia). Anosmia is loss of sense of smell. Hypochlorhydria, a decrease in stomach acid, does not affect swallowing. Salivary gland tumors are not common.

Which digestive substances are active or activated in the stomach (select all that apply)? a. Bile b. Pepsin c. Gastrin d. Maltase e. Secretin f. Amylase

Correct answer: b, c Rationale: Pepsinogen is changed to pepsin by acidity of the stomach, where it begins to break down proteins. Gastrin stimulates gastric acid secretion and motility and maintains lower esophageal sphincter tone. The stomach also secretes lipase for fat digestion.Bile is secreted by the liver and stored in the gallbladder for emulsifying fats. Maltase is secreted in the small intestine and converts maltose to glucose. Secretin is secreted y the duodenal mucosa and inhibits gastric motility and acid secretion. Amylase is secured in the small intestine and by the pancreas for carbohydrate digestion.

The nurse should recognize that the liver performs which functions (select all that apply) a. Bile storage b. Detoxification c. Protein metabolism d. Steroid metabolism e. Red blood cell (RBC) destruction

Correct answer: b, c, d Rationale: The liver performs multiple major functions that aid in the maintenance of homeostasis. These include metabolism of proteins and steroids as well as detoxification of drugs and metabolic waste products. The Kupffer cells of the liver participate in the breakdown of old RBCs. The liver produces bile, but storage occurs in the gall bladder.

The patient tells the nurse she had a history of abdominal pain, so she had a surgery to make an opening into the common bile duct to remove stones. The nurse knows that this surgery is called a a. colectomy b. cholecystectomy c. choledocholithotomy d. choledochojejunostomy

Correct answer: c Rationale: A choledocholithotomy is an opening into the common bile duct for the removal of stones. A colectomy is the removal of the colon. The cholecystectomy is the removal of the gallbladder. The choledochojejunostomy is an opening between the common bile duct and the jejunum.

What is a normal finding during physical assessment of the mouth? a. A red, slick appearance of the tongue b. Uvular deviation to the side on saying "Ahh" c. A thin, white coating of the dorsum of the tongue d. Scattered red, smooth areas on the dorsum of the tongue

Correct answer: c Rationale: A thin white coating of the dorsum (top) of the tongue is normal. A red, slick appearance is characteristic of cobalamin deficiency and scattered red, smooth areas on the tongue are known as geographic tongue. The uvula should remain in the midline while the patient is saying "Ahh"

When assessing a patient's abdomen, what would be most appropriate for the nurse to do? a. Palpate the abdomen before auscultation. b. Percuss the abdomen before auscultation. c. Auscultate the abdomen before palpation. d. Perform deep palpation before light palpation.

Correct answer: c Rationale: During examination of the abdomen, auscultation is done before percussion and palpation because these latter procedures may alter the bowel sounds.

The health care team is assessing a male patient for acute pancreatitis after he presented to the emergency department with severe abdominal pain. Which laboratory value is the best diagnostic indicator of acute pancreatitis? a. Gastric pH b. Blood glucose c. Serum amylase d. Serum potassium

Correct answer: c Rationale: Elevated serum amylase levels indicate early pancreatic dysfunction and are used to diagnose acute pancreatitis. Serum lipase levels stay elevated longer than serum amylase in acute pancreatitis. Blood glucose, gastric pH, and potassium levels are not direct indicators of acute pancreatic dysfunction.

A patient who is scheduled for surgery with general anesthesia in 1 hour is observed with a moist, but empty water glass in his hand. Which assessment finding may indicate that the patient drank a glass of water? a. Flat abdomen without movement upon inspection b. Tenderness at left upper quadrant upon palpation c. Easily heard, loud gurgling in the right upper quadrant d. High-pitched, hollow sounds in the left upper quadrant

Correct answer: c Rationale: If the patient drank water on an empty stomach, gurgling can be assessed without a stethoscope or assessed with auscultation. High-pitched, hollow sounds are tympanic and indicate an empty cavity. A flat abdomen and tenderness do not indicate that the patient drank a glass of water.

When caring for the patient with heart failure, the nurse knows that which gastrointestinal process is most dependent on cardiac output and may affect the patient's nutritional status? a. Ingestion b. Digestion c. Absorption d. Elimination

Correct answer: c Rationale: Substances that interface with the absorptive surfaces of the GI tract (primarily in the small intestine) diffuse across the intestinal membranes into intestinal capillaries and are then carried to other parts of the body for use in energy production. The cardiac output provides the blood flow for this absorption of nutrients to occur.

How will an obstruction at the ampulla of Vater affect the digestion of all nutrients? a. Bile is responsible for emulsification of all nutrients and vitamins. b. Intestinal digestive enzymes are released through the ampulla of Vater. c. Both bile and pancreatic enzymes enter the duodenum at the ampulla of Vater. d. Gastric contents can ply pass to the duodenum when the ampulla of Vater is open.

Correct answer: c Rationale: The ampulla of Vater is the site where the pancreatic duct and common bile duct enter the duodenum and the opening and closing of the ampulla is controlled by the sphincter of Oddi. Because bile from the common bile duct is needed for emulsification of fat to promote digestion and pancreatic enzymes from the pancreas are needed for digestion of all nutrients, a blockage at this point would affect the digestion of all nutrients. Gastric contents pass into the duodenum through the pylorus or pyloric valve.

A patient's serum liver enzyme tests reveal an elevated aspartate aminotransferase (AST). The nurse recognizes what about the elevated AST? a. It eliminates infection as a cause of liver damage. b. It is diagnostic for liver inflammation and damage. c. Tissue damage in organs other than the liver may be identified. d. Nervous system symptoms related to hepatic encephalopathy may be the cause.

Correct answer: c Rationale: The aspartate aminotransferase (AST) level is elevated in liver disease but it is important to note that it is also elevated in damage to the heart and lungs and is not a specific test for liver function. Measurements of most of the transaminases involves nonspecific tests unless isoenzyme fractions are determined. Hepatic encephalopathy is related to elevated ammonia levels.

The nurse is assessing a 50-year-old woman admitted with a possible bowel obstruction. Which assessment finding would be expected in this patient? a. Tympany to abdominal percussion b. Aortic pulsation visible in epigastric region c. High-pitched sounds on abdominal auscultation d. Liver border palpable 1 cm below the right costal margin

Correct answer: c Rationale: The bowel sounds are more high pitched (rushes and tinkling) when the intestines are under tension, as in intestinal obstruction. Bowel sounds may also be diminished or absent with an intestinal obstruction. Normal findings include aortic pulsations on inspection and tympany with percussion, and the liver may be palpable 1 to 2 cm along the right costal margin.

A patient receives atropine, an anticholinergic drug, in preparation for surgery. The nurse expects this drug to affect the GI tract by doing what? a. Increasing gastric emptying b. Relaxing pyloric and ileocecal sphincters c. Decreasing secretions and peristaltic action d. Stimulation the nervous system of the GI tract

Correct answer: c Rationale: The parasympathetic nervous system stimulates activity of the gastrointestinal (GI) tract, increasing motility and secretions and relaxing sphincters to promote movement of contents. A drug that blocks this activity decreases secretions and peristalsis, slows gastric emptying, and contracts sphincters. The enteric nervous system of the GI tract is modulated by sympathetic and parasympathetic influence.

Priority Decision: When caring for a patient who has had most of the stomach surgically removed, what is important for the nurse to teach the patient? a. Extra iron will need to be taken to prevent anemia. b. Avoid foods with lactose to prevent bloating and diarrhea. c. Lifelong supplementation of cobalamin (vitamin B12) will be needed. d. Because of the absence of digestive enzymes, protein malnutrition is likely.

Correct answer: c Rationale: The stomach secretes intrinsic factor, necessary for cobalamin (vitamin B12) absorption in the intestine. When part or all of the stomach is removed, cobalamin must be supplemented for life. The other options will not be a problem.

Which nursing actions are indicated for a liver biopsy (select all that apply)? a. Observe for white stools b. Monitor for rectal bleeding c. Monitor for internal bleeding d. Position to right side after test e. Ensure bowel preparation was done f. Check coagulation status before test

Correct answer: c, d, f Rationale: Because the liver is a vascular organ, vital signs are monitored to assess for internal bleeding. Prevention of bleeding is the reason for positioning on the right side for at least 2 hours and for splinting the puncture site. Again, because of the vasculature of the liver, coagulation status is checked before the biopsy is done. White stools occur with upper gastrointestinal (UGI) or barium swallow tests. No smoking is to be done after midnight before the study with an UGI. The bowel must be cleared before a lower GI or barium enema, a virtual colonoscopy, or a colonoscopy. Rectal bleeding may occur with a sigmoidoscopy or colonoscopy. A perforation may occur with an esophagogastroduodenoscopy (EGD), ERCP, or peritoneoscopy.

The nurse is performing a focused abdominal assessment of a patient who has been recently admitted. In order to palpate the patient's liver, where should the nurse palpate the patient's abdomen? a. Left lower quadrant b. Left upper quadrant c. Right lower quadrant d. Right upper quadrant

Correct answer: d Rationale: Although the left lobe of the liver is located in the left upper quadrant of the abdomen, the bulk of the liver is located in the right upper quadrant.

After eating, a patient with an inflamed gallbladder experiences pain caused by contraction of the gallbladder. What is the mechanism responsible for this action? a. Production of bile by the liver b. Production of secretin by the duodenum c. Release of gastrin from the stomach antrum d. Production of cholecystokinin by the duodenum

Correct answer: d Rationale: Cholecystokinin is secreted by the duodenal mucosa when fats and amino acids enter the duodenum and stimulate the gallbladder to release bile to emulsify the fats for digestion. The bile is produced by the liver but stored in the gallbladder. Secretin is responsible for stimulating pancreatic bicarbonate secretion and gastrin increases gastric motility and acid secretion.

The nurse is reviewing the home medication list for a 44-year-old man admitted with suspected hepatic failure. Which medication could cause hepatotoxicity? a. Nitroglycerin b. Digoxin (Lanoxin) c. Ciprofloxacin (Cipro) d. Acetaminophen (Tylenol)

Correct answer: d Rationale: Many chemicals and drugs are potentially hepatotoxic (see Table 39-6) and result in significant patient harm unless monitored closely. For example, chronic high doses of acetaminophen and nonsteroidal antiinflammatory drugs (NSAIDs) may be hepatotoxic.

What is a clinical manifestation of age-related changes in the GI system that the nurse may find in an older patient? a. Gastric hyperacidity b. Intolerance to fatty foods c. Yellowish tinge to the skin d. Reflux of gastric contents into the esophagus

Correct answer: d Rationale: There is decreased tone of the lower esophageal sphincter with again and regurgitation of gastric contents back into the esophagus occurs, causing heartburn and belching. There is a decrease in hydrochloric acid secretion with aging. Jaundice and intolerance to fatty foods are symptoms of liver or gallbladder disease and are not normal age-related findings.

A 68-year-old patient is in the office for a physical. She notes that she no longer has regular bowel movements. Which suggestion by the nurse would be most helpful to the patient? a. Take an additional laxative to stimulate defecation. b. Eat less acidic foods to enable the gastrointestinal system to increase peristalsis. c. Eat less food at each meal to prevent feces from backing up related to slowed peristalsis. d. Attempt defecation after breakfast because gastrocolic reflexes increase colon peristalsis at that time.

Correct answer: d Rationale: When food inters the stomach and duodenum, the gastrocolic and duodenocolic reflexes are initiated and are more active after the first daily meal. Additional laxatives or laxative abuse contribute to constipation in older adults. Decreasing food intake is not recommended, as many older adults have a decreased appetite. Fibre and fluids should be increased.

The ED nurse has inspected, auscultated, and palpated the abdomen with no obvious abnormalities, except pain. When the nurse palpates the abdomen for rebound tenderness, there is severe pain. The nurse should know that this could indicate what problem? a. Hepatic cirrhosis b. Hypersplenomegaly c. Gall bladder distention d. Peritoneal inflammation

Correct answer: d Rationale: When palpating for rebound tenderness, the problem area of the abdomen will produce pain and severe muscle spasm when there is peritoneal inflammation. Hepatic cirrhosis, hypersplenomegaly, and gall bladder distention do not manifest with rebound tenderness.

When teaching the patient about the diet for diverticular disease, which foods should the nurse recommend? A) White bread, cheese, and green beans B) Fresh tomatoes, pears, and corn flakes C) Oranges, baked potatoes, and raw carrots D) Dried beans, All Bran (100%) cereal, and raspberries

D) A high fiber diet is recommended for diverticular disease. Dried beans, All Bran (100%) cereal, and raspberries all have higher amounts of fiber than white bread, cheese, green beans, fresh tomatoes, pears, corn flakes, oranges, baked potatoes, and raw carrots.

When planning care for a patient with cirrhosis, the nurse will give highest priority to which of the following nursing diagnoses? A. Imbalanced nutrition: less than body requirements B. Impaired skin integrity related to edema, ascites, and pruritus C. Excess fluid volume related to portal hypertension and hyperaldosteronism D. Ineffective breathing pattern related to pressure on diaphragm and reduced lung volume

D) Although all of these nursing diagnoses are appropriate and important in the care of a patient with cirrhosis, airway and breathing are always the highest priorities.

The nurse is performing a focused abdominal assessment of a patient who has been recently admitted. In order to palpate the patient's liver, where should the nurse palpate the patient's abdomen? A) Left lower quadrant B) Left upper quadrant C) Right lower quadrant D) Right upper quadrant

D) Although the left lobe of the liver is located in the left upper quadrant of the abdomen, the bulk of the liver is located in the right upper quadrant.

The nurse is preparing to administer a dose of bisacodyl (Dulcolax). In explaining the medication to the patient, the nurse would state that it acts in which of the following ways? A) Increases bulk in the stool B) Lubricates the intestinal tract to soften feces C) Increases fluid retention in the intestinal tract D) Increases peristalsis by stimulating nerves in the colon wall

D) Bisacodyl is a stimulant laxative that aids in producing a bowel movement by irritating the colon wall and stimulating enteric nerves. It is available in oral and suppository forms. Fiber and bulk forming drugs increase bulk in the stool; water and stool softeners soften feces, and saline and osmotic solutions cause fluid retention in the intestinal tract.

The nurse is preparing to administer a scheduled dose of docusate sodium (Colace) when the patient complains of an episode of loose stool and does not want to take the medication. Which of the following is the appropriate action by the nurse? A) Write an incident report about this untoward event. B) Attempt to have the family convince the patient to take the ordered dose. C) Withhold the medication at this time and try to administer it later in the day. D) Chart the dose as not given on the medical record and explain in the nursing progress notes.

D) Bisacodyl is a stimulant laxative that aids in producing a bowel movement by irritating the colon wall and stimulating enteric nerves. It is available in oral and suppository forms. Fiber and bulk forming drugs increase bulk in the stool; water and stool softeners soften feces, and saline and osmotic solutions cause fluid retention in the intestinal tract.

The patient with a history of lung cancer and hepatitis C has developed liver failure and is considering liver transplantation. After the comprehensive evaluation, the nurse knows that which factor discovered may be a contraindication for liver transplantation? A) Has completed a college education B) Has been able to stop smoking cigarettes C) Has well-controlled type 1 diabetes mellitus D) The chest x-ray showed another lung cancer lesion.

D) Contraindications for liver transplant include severe extrahepatic disease, advanced hepatocellular carcinoma or other cancer, ongoing drug and/or alcohol abuse, and the inability to comprehend or comply with the rigorous post-transplant course.

What should the nurse instruct the patient to do to best enhance the effectiveness of a daily dose of docusate sodium (Colace)? A) Take a dose of mineral oil at the same time. B) Add extra salt to food on at least one meal tray. C) Ensure dietary intake of 10 g of fiber each day. D) Take each dose with a full glass of water or other liquid.

D) Docusate lowers the surface tension of stool, permitting water and fats to penetrate and soften the stool for easier passage. The patient should take the dose with a full glass of water and should increase overall fluid intake, if able, to enhance effectiveness of the medication. Dietary fiber intake should be a minimum of 20 g daily to prevent constipation. Mineral oil and extra salt are not recommended.

The nurse is caring for a 68-year-old patient admitted with abdominal pain, nausea, and vomiting. The patient has an abdominal mass, and a bowel obstruction is suspected. The nurse auscultating the abdomen listens for which type of bowel sounds that are consistent with the patient's clinical picture? A) Low-pitched and rumbling above the area of obstruction B) High-pitched and hypoactive below the area of obstruction C) Low-pitched and hyperactive below the area of obstruction D) High-pitched and hyperactive above the area of obstruction

D) Early in intestinal obstruction, the patient's bowel sounds are hyperactive and high-pitched, sometimes referred to as "tinkling" above the level of the obstruction. This occurs because peristaltic action increases to "push past" the area of obstruction. As the obstruction becomes complete, bowel sounds decrease and finally become absent.

The nurse is caring for a postoperative patient with a colostomy. The nurse is preparing to administer a dose of famotidine (Pepcid) when the patient asks why the medication was ordered since the patient does not have a history of heartburn or gastroesophageal reflux disease (GERD). What response by the nurse would be the most appropriate? A) "This will prevent air from accumulating in the stomach, causing gas pains." B) "This will prevent the heartburn that occurs as a side effect of general anesthesia." C) "The stress of surgery is likely to cause stomach bleeding if you do not receive it." D) "This will reduce the amount of HCl in the stomach until the nasogastric tube is removed and you can eat a regular diet again."

D) Famotidine is an H2-receptor antagonist that inhibits gastric HCl secretion and thus minimizes damage to gastric mucosa while the patient is not eating a regular diet after surgery. Famotidine does not prevent air from accumulating in the stomach or stop the stomach from bleeding. Heartburn is not a side effect of general anesthesia.

The health care provider orders lactulose for a patient with hepatic encephalopathy. The nurse will monitor for effectiveness of this medication for this patient by assessing which of the following? A. Relief of constipation B. Relief of abdominal pain C. Decreased liver enzymes D. Decreased ammonia levels

D) Hepatic encephalopathy is a complication of liver disease and is associated with elevated serum ammonia levels. Lactulose traps ammonia in the intestinal tract. Its laxative effect then expels the ammonia from the colon, resulting in decreased serum ammonia levels and correction of hepatic encephalopathy.

The family of a patient newly diagnosed with hepatitis A asks the nurse what they can do to prevent becoming ill themselves. Which of the following responses by the nurse is most appropriate? A. "The hepatitis vaccine will provide immunity from this exposure and future exposures." B. "I am afraid there is nothing you can do since the patient was infectious before admission." C. "You will need to be tested first to make sure you don't have the virus before we can treat you." D. "An injection of immunoglobulin will need to be given to prevent or minimize the effects from this exposure."

D) Immunoglobulin provides temporary (1-2 months) passive immunity and is effective for preventing hepatitis A if given within 2 weeks after exposure. It may not prevent infection in all persons, but it will at least modify the illness to a subclinical infection. The hepatitis vaccine is only used for preexposure prophylaxis

The nurse would question the use of which cathartic agent in a patient with renal insufficiency? A) Bisacodyl (Dulcolax) B) Lubiprostone (Amitiza) C) Cascara sagrada (Senekot) D) Magnesium hydroxide (Milk of Magnesia)

D) Milk of Magnesia may cause hypermagnesemia in patients with renal insufficiency. The nurse should question this order with the health care provider. Bisacodyl, lubiprostone, and cascara sagrada are safe to use in patients with renal insufficiency as long as the patient is not currently dehydrated.

A pt reports having dry mouth and asks for some liquid to drink. The nurse reasons that this symptom can most likely be attributed to a common adverse effect of which of the following medications? A) Digoxin (Lanoxin) B) Cefotetan (Cefotan) C) Famotidine (Pepcid) D) Promethazine (Phenergan)

D) Promethazine (Phenergan) A common adverse effect of promethazine, an antihistamine antiemetic agent, is dry mouth; another is blurred vision.

A patient who has hepatitis B surface antigen (HBsAg) in the serum is being discharged with pain medication after knee surgery. Which medication order should the nurse question because it is most likely to cause hepatic complications? A) Tramadol (Ultram) B) Hydromorphone (Dilaudid) C) Oxycodone with aspirin (Percodan) D) Hydrocodone with acetaminophen (Vicodin)

D) The analgesic with acetaminophen should be questioned because this patient is a chronic carrier of hepatitis B and is likely to have impaired liver function. Acetaminophen is not suitable for this patient because it is converted to a toxic metabolite in the liver after absorption, increasing the risk of hepatocellular damage.

The patient with suspected pancreatic cancer is having many diagnostic studies done. Which one can be used to establish the diagnosis of pancreatic adenocarcinoma and for monitoring the response to treatment? A) Spiral CT scan B) A PET/CT scan C) Abdominal ultrasound D) Cancer-associated antigen 19-9

D) The cancer-associated antigen 19-9 (CA 19-9) is the tumor marker used for the diagnosis of pancreatic adenocarcinoma and for monitoring the response to treatment. Although a spiral CT scan may be the initial study done and provides information on metastasis and vascular involvement, this test and the PET/CT scan or abdominal ultrasound do not provide additional information.

When teaching the patient with acute hepatitis C (HCV), the patient demonstrates understanding when the patient makes which statement? A) "I will use care when kissing my wife to prevent giving it to her." B) "I will need to take adofevir (Hepsera) to prevent chronic HCV." C) "Now that I have had HCV, I will have immunity and not get it again." D) "I will need to be checked for chronic HCV and other liver problems."

D) The majority of patients who acquire HCV usually develop chronic infection, which may lead to cirrhosis or liver cancer. HCV is not transmitted via saliva, but percutaneously and via high-risk sexual activity exposure. The treatment for acute viral hepatitis focuses on resting the body and adequate nutrition for liver regeneration. Adofevir (Hepsera) is taken for severe hepatitis B (HBV) with liver failure. Chronic HCV is treated with pegylated interferon with ribavirin. Immunity with HCV does not occur as it does with HAV and HBV, so the patient may be reinfected with another type of HCV.

The nurse asks a 68-year-old patient scheduled for colectomy to sign the operative permit as directed in the physician's preoperative orders. The patient states that the physician has not really explained very well what is involved in the surgical procedure. What is the most appropriate action by the nurse? A) Ask family members whether they have discussed the surgical procedure with the physician. B) Have the patient sign the form and state the physician will visit to explain the procedure before surgery. C) Explain the planned surgical procedure as well as possible and have the patient sign the consent form. D) Delay the patient's signature on the consent and notify the physician about the conversation with the patient.

D) The patient should not be asked to sign a consent form unless the procedure has been explained to the satisfaction of the patient. The nurse should notify the physician, who has the responsibility for obtaining consent.

The nurse is teaching a group of high school students about the prevention of food poisoning. Which comment by the student shows understanding of foodborne illness protection? A) "We like to mix up the ingredients so the flavors will melt before we cook our beef stew." B) "For a snack, I like to eat raw cookie dough from the package instead of baking the cookies." C) "We only have one cutting board, so we cut up our chicken and salad vegetables at the same time." D) "When they gave me a pink hamburger I sent it back and asked for a new bun and clean plate."

D) The student who did not accept the pink hamburger and asked for a new bun and clean plate understood that the pink meat may not have reached 160° and could be contaminated with bacteria. Mixing ingredients and leaving them long enough for the flavors to melt, eating raw cookie dough from a refrigerated package, and only using one cutting board without washing it with hot soapy water between the chicken and salad vegetables could all lead to food poisoning from contamination.

The ED nurse has inspected, auscultated, and palpated the abdomen with no obvious abnormalities, except pain. When the nurse palpates the abdomen for rebound tenderness, there is severe pain. The nurse should know that this could indicate what problem? A) Hepatic cirrhosis B) Hypersplenomegaly C) Gall bladder distention D) Peritoneal inflammation

D) When palpating for rebound tenderness, the problem area of the abdomen will produce pain and severe muscle spasm when there is peritoneal inflammation. Hepatic cirrhosis, hypersplenomegaly, and gall bladder distention do not manifest with rebound tenderness.

The nurse is preparing to administer a scheduled dose of docusate sodium (Colace) when the patient reports an episode of loose stool and does not want to take the medication. What is the appropriate action by the nurse? A) Write an incident report about this untoward event. B) Attempt to have the family convince the patient to take the ordered dose. C) Withhold the medication at this time and try to administer it later in the day. D) Chart the dose as not given on the medical record and explain in the nursing progress notes.

D) Whenever a patient refuses medication, the dose should be charted as not given with an explanation of the reason documented in the nursing progress notes. In this instance, the refusal indicates good judgment by the patient, and the patient should not be encouraged to take it today.

A 26-year-old patient is looking down as she tells the nurse that she is afraid to use the treatment recommended for her psoriasis because her mother had a lot of problems with all the creams she used to try to treat her psoriasis. How should the nurse respond to the patient? A. "You will only know if you try it and see." B. "You may need to get counseling to help you cope." C. "No treatment is medically necessary, but it can be removed." D. "Topical, light therapy, and systemic medications are now available."

D. "Topical, light therapy, and systemic medications are now available." Treatment of psoriasis usually involves a combination of strategies including topical treatments, phototherapy, and/or systemic medications including biologic drugs. Telling her that she will only know if she tries or that she may need counseling is denying the patient's concern. Psoriasis is treated to manage the disease as the patient may have a weakened immune system and be at risk for cardiovascular disease.

To the nurse, a patient describes small, firm, reddened raised lesions with flat, rough patches that are causing intense pruritus. What should be the nurse's next assessment? A. History of seasonal allergies B. Initiation of new medication C. Previous pruritic skin lesions D. Activities in past 2 to 7 days

D. Activities in past 2 to 7 days The patient's lesions are papules and plaques characteristic of contact dermatitis. The nurse should ask the patient about activities over the past 2 to 7 days to identify potential allergens because contact dermatitis has a delayed onset. Even if an offending agent is not identified, the nurse can provide patient teaching about managing the pruritus and preventing infection by decreasing scratching. Seasonal allergies and new medications are more likely to cause urticaria than papules and plaque. The nurse should also ask about pruritic rashes in the past to determine potential illnesses that can cause dermatologic manifestations.

What practice should the nurse teach a patient to follow when the patient is applying topical medication? A. Avoid applying medications directly on to dressings. B. Use a tongue blade whenever the patient's skin integrity allows. C. Avoid covering skin regions that have topical medication in place. D. Apply a layer of medication that is just thick enough to ensure coverage.

D. Apply a layer of medication that is just thick enough to ensure coverage. Patients should be directed to avoid applying topical medications too thickly. Medications may be applied directly on to dressings, and regions with medications may be covered. A tongue blade is not normally used for the application of a thin coat.

The nurse is teaching the residents of an independent living facility about preventing skin infections and infestations. What should be included in the teaching? A. Use cool compresses if an infection occurs. B. Oral antibiotics will be needed for any skin changes. C. Antiviral agents will be needed to prevent outbreaks. D. Inspect skin for changes when bathing with mild soap.

D. Inspect skin for changes when bathing with mild soap. Individuals living in independent living facilities are usually older, which means their skin does not need cleaning with hot water and vigorous scrubbing or as often as a younger person. Mild soap (e.g., Ivory) should be used to avoid loss of protection from neutralization of the skin's surface. The skin should be inspected for changes with bathing. Cool compresses are used with ringworm or stings for the antiinflammatory effect. Oral antibiotics are used for Lyme disease from ticks. Antiviral agents are used for viral infections but not to prevent outbreaks.

5. A nurse is providing teaching to a client who is scheduled for a phentolamine blocking test. This test supports a diagnosis for which of the following disorders? A. Addison's disease B. Diabetes mellitus C. Cushing's disease D. Pheochromocytoma

D. Pheochromocytoma

The nurse suspects that a client with DM may be non-compliant with the treatment plan. The nurse should know that a reliable test to evaluate if the client is routinely compliant with the prescribed regimen is what?

Glucosylated hemoglobin levels or Hemoglobin A1C

ANS: A The treatment for impetigo includes softening of the crusts with warm saline soaks and then soap-and-water removal. Alcohol-based cleansers and use of petroleum jelly are not recommended for impetigo. Antibiotic ointments, such as mupirocin (Bactroban), may be applied to the lesions.

The health care provider diagnoses impetigo in a patient who has crusty vesicopustular lesions on the lower face. Which instructions should the nurse include in the teaching plan? a. Clean the infected areas with soap and water. b. Apply alcohol-based cleansers on the lesions. c. Avoid use of antibiotic ointments on the lesions. d. Use petroleum jelly (Vaseline) to soften crusty areas.

ANS: D Topical 5-FU causes an initial reaction of erythema, itching, and erosion that lasts 4 weeks after application of the medication is stopped. The medication is topical, so there are no systemic effects such as increased infection risk, anorexia, or nausea.

The health care provider prescribes topical 5-FU for a patient with actinic keratosis on the left cheek. The nurse should include which statement in the patient's instructions? a. "5-FU will shrink the lesion so that less scarring occurs once the lesion is excised." b. "You may develop nausea and anorexia, but good nutrition is important during treatment." c. "You will need to avoid crowds because of the risk for infection caused by chemotherapy." d. "Your cheek area will be painful and develop eroded areas that will take weeks to heal."

ANS: D Pale, cool skin indicates a possible decrease in circulation, so the surgeon should be notified immediately. The other assessment data indicate a need for ongoing assessment or nursing action. A heart rate of 110 beats/minute may be related to the stress associated with surgery. Assessment of other vital signs and continued monitoring are appropriate. Because local anesthesia would be used for the procedure, numbness of the incisional area is expected immediately after surgery. The nurse should monitor for return of feeling.

The nurse assesses a patient who has just arrived in the postanesthesia recovery area (PACU) after a blepharoplasty. Which assessment data should be reported to the surgeon immediately? a. The patient complains of incisional pain. b. The patient's heart rate is 110 beats/minute. c. The patient is unable to detect when the eyelids are touched. d. The skin around the incision is pale and cold when palpated.

ANS: C Creams and ointments should be applied in a thin layer to avoid wasting the medication. The other actions by the patient indicate that the teaching has been successful

The nurse instructs a patient about application of corticosteroid cream to an area of contact dermatitis on the right leg. Which patient action indicates that further teaching is needed? a. The patient takes a tepid bath before applying the cream. b. The patient spreads the cream using a downward motion. c. The patient applies a thick layer of the cream to the affected skin. d. The patient covers the area with a dressing after applying the cream.

ANS: D Cleaning the skin is within the education and scope of practice for UAP. Administration of medication, obtaining cultures, and evaluation are higher-level skills that require the education and scope of practice of licensed nursing personnel.

The nurse is caring for a patient diagnosed with furunculosis. Which nursing action could the nurse delegate to unlicensed assistive personnel (UAP)? a. Applying antibiotic cream to the groin. b. Obtaining cultures from ruptured lesions. c. Evaluating the patient's personal hygiene. d. Cleaning the skin with antimicrobial soap.

ANS: B Neosporin can cause contact dermatitis. The other medications are being used appropriately by the patient.

The nurse is interviewing a patient with contact dermatitis. Which finding indicates a need for patient teaching? a. The patient applies corticosteroid cream to pruritic areas. b. The patient uses Neosporin ointment on minor cuts or abrasions. c. The patient adds oilated oatmeal (Aveeno) to the bath water every day. d. The patient takes diphenhydramine (Benadryl) at night if itching occurs.

ANS: D The presence of acanthosis nigricans in skinfolds suggests either having type 2 diabetes or being at an increased risk for it. The description of the patient's skin does not indicate problems with fungal infection, poor hygiene, or the need to dry the skinfolds better.

The nurse notes darker skin pigmentation in the skinfolds of a middle-aged patient who has a body mass index of 40 kg/m2. What is the nurse's best action? a. Teach the patient about the treatment of fungal infection. b. Discuss the use of drying agents to minimize infection risk. c. Instruct the patient about the use of mild soap to clean skinfolds. d. Ask the patient about type 2 diabetes or if there is a family history of it.

ANS: C The appearance of the lesions is consistent with an oral candidiasis (thrush) infection, which can occur in patients who are taking medications such as immunosuppressants or antibiotics. Candidiasis is not associated with poor oral hygiene or lower respiratory infections. The lesions do not look like an oral herpes infection.

The nurse notes the presence of white lesions that resemble milk curds in the back of a patient's throat. Which question by the nurse is appropriate at this time? a. "Do you have a productive cough?" b. "How often do you brush your teeth?" c. "Are you taking any medications at present?" d. "Have you ever had an oral herpes infection?"

ANS: A Because isotretinoin is teratogenic, contraception is required for women who are using this medication. The nurse will need to determine whether the patient is using other birth control methods. More information about the other patient data may also be needed, but the other data do not indicate contraindications to isotretinoin use.

The nurse working in the dermatology clinic assesses a young adult female patient who is taking isotretinoin (Accutane) to treat severe cystic acne. Which assessment finding is most indicative of a need for further questioning of the patient? a. The patient recently had an intrauterine device removed. b. The patient already has some acne scarring on her forehead. c. The patient has also used topical antibiotics to treat the acne. d. The patient has a strong family history of rheumatoid arthritis.

ANS: A The description of the lesion is consistent with possible malignant melanoma. This patient should be assessed as soon as possible by the health care provider. Itching is common after using topical fluorouracil and redness is an expected finding a few days after a chemical peel. Skin tags are common, benign lesions after midlife.

There is one opening in the schedule at the dermatology clinic, and 4 patients are seeking appointments today. Which patient will the nurse schedule for the available opening? a. 38-year old with a 7-mm nevus on the face that has recently become darker b. 62-year-old with multiple small, soft, pedunculated papules in both axillary areas c. 42-year-old with complaints of itching after using topical fluorouracil on the nose d. 50-year-old with concerns about skin redness after having a chemical peel 3 days ago.

ANS: D Careful hand washing and the safe disposal of soiled dressings are the best means of preventing the spread of skin problems. Sterile glove and sterile saline use during wound care will not necessarily prevent spread of infection. Applying antibiotic ointment will treat the bacteria but not necessarily prevent the spread of infection.

What is the best method to prevent the spread of infection when the nurse is changing the dressing over a wound infected with Staphylococcus aureus? a. Change the dressing using sterile gloves. b. Soak the dressing in sterile normal saline. c. Apply antibiotic ointment over the wound. d. Wash hands and properly dispose of soiled dressings.

ANS: C The patient has clinical manifestations that could be caused by systemic problems such as malnutrition or hypothyroidism, so further diagnostic evaluation is indicated. Patient teaching about nutrition, addressing the patient's dry skin, and referral to a podiatrist may also be needed, but the priority is to rule out underlying disease that may be causing these manifestations.

When assessing a new patient at the outpatient clinic, the nurse notes dry, scaly skin; thin hair; and thick, brittle nails. What is the nurse's best action? a. Instruct the patient about the importance of nutrition in skin health. b. Make a referral to a podiatrist so that the nails can be safely trimmed. c. Consult with the health care provider about the need for further diagnostic testing. d. Teach the patient about using moisturizing creams and lotions to decrease dry skin.

ANS: A The patient should stay out of the sun. If that is not possible, teach them to wear sunscreen when taking medications that can cause photosensitivity. The other statements are not accurate.

Which information should the nurse include when teaching a patient who has just received a prescription for ciprofloxacin (Cipro) to treat a urinary tract infection? a. Use a sunscreen with a high SPF when exposed to the sun. b. Sun exposure may decrease the effectiveness of the medication. c. Photosensitivity may result in an artificial-looking tan appearance. d. Wear sunglasses to avoid eye damage while taking this medication.

ANS: D The risk for skin damage from the sun is highest with exposure between 10 AM and 2 PM. No sunscreen is completely water resistant. Sunscreens classified as water resistant sunscreens still need to be reapplied after swimming. Sunscreen with an SPF of at least 15 is recommended for people at normal risk for skin cancer. Although gradually increasing sun exposure may decrease the risk for burning, the risk for skin cancer is not decreased.

Which information should the nurse include when teaching patients about decreasing the risk for sun damage to the skin? a. Use a sunscreen with an SPF of at least 8 to 10 for adequate protection. b. Water resistant sunscreens will provide good protection when swimming. c. Increase sun exposure by no more than 10 minutes a day to avoid skin damage. d. Try to stay out of the sun between the hours of 10 AM and 2 PM (regular time)

ANS: C Warm water and moisturizing soap will avoid overdrying the skin. Because older patients have dryer skin, daily bathing and shampooing are not necessary and may dry the skin unnecessarily. Antibacterial soaps are not necessary. Lotions should be applied while the skin is still damp to seal moisture in.

Which information will the nurse include when teaching an older patient about skin care? a. Dry the skin thoroughly before applying lotions. b. Bathe and wash hair daily with soap and shampoo. c. Use warm water and a moisturizing soap when bathing. d. Use antibacterial soaps when bathing to avoid infection.

The patient has diabetes mellitus and chronic obstructive pulmonary disease that has been treated with high-dose corticosteroids for the past several years. Which dermatologic manifestations could be related to these systemic problems (select all that apply)? a. Acne b. Increased sweating c. Dry, coarse, brittle hair d. Impaired wound healing e. Erythematous plaques of the shins f. Decreased subcutaneous fat over extremities

a, d, e, f. Glucocorticoid excess can cause acne and decreased subcutaneous fat over the extremities. Diabetes mellitus can cause erythematous plaques of the shins and both the corticosteroids and diabetes can impair or delay wound healing. Increased sweating is seen with hyperthyroidism and coarse, brittle hair is seen with hypothyroidism

Which statements are true about skin and skin care (select all that apply)? a. One of the detrimental effects of obesity on the skin is increased sweating. b. The nutrient that is critical in maintaining and repairing the structure of epithelial cells is vitamin C. c. Exposure to UVA rays is believed to be the most important factor in the development of skin cancer. d. The photosensitivity caused by various drugs can be blocked by the use of topical hydrocortisone. e. Photosensitivity results when certain chemicals in body cells and tissues absorb light from the sun and release energy that harms the tissues and cells. f. When teaching a patient about the use of sunscreens that protect against exposure to both UVA and UVB rays, the nurse advises the patient to look for the inclusion of benzophenones.

a, e, f. Vitamin A, not vitamin C, is critical in maintaining and repairing the structure of epithelial cells. Exposure to UVB rays, not UVA rays, is believed to be the most important factor in the development of skin cancer. Sunscreen, not topical hydrocortisone, can block the photosensitivity caused by various drugs.

What skin condition has keratotic and firm lesions, is a precursor of squamous cell carcinoma, and is treated with topical fluorouracil (5-FU)? a. Actinic keratosis b. Basal cell carcinoma c. Malignant melanoma d. Squamous cell carcinoma

a. Basal cell carcinoma is noduloulcerative with pearly borders. Malignant melanoma tumors arise in melanocytes. Malignant melanoma is the deadliest skin cancer and has an increased risk in people with dysplastic nevus syndrome. Squamous cell carcinoma is a malignant neoplasm of keratinizing epidermal cells.

What are the most appropriate dressings to use to promote comfort for a patient with an inflamed, pruritic dermatitis? a. Cool tap water dressings b. Cool acetic acid dressings c. Warm sterile saline dressings d. Warm potassium permanganate dressings

a. Dressings used to treat pruritic lesions should be cool to cause vasoconstriction and to have an antiinflammatory effect. Water is most commonly used and it does not need to be sterile. Acetic acid solutions are bacteriocidal and are used to treat skin infections.

What is the most common reason elective cosmetic surgery is requested by patients? a. Improve self-image c. Lighten the skin in pigmentation problems b. Remove deep acne scars d. Prevent skin changes associated with aging

a. Improvement of body image is the most common reason for undergoing cosmetic surgery; appearance is an important part of confidence and self-assurance. Acne scars, pigmentation problems, and wrinkling can be treated with cosmetic surgery but the surgery does not prevent the skin changes associated with aging.

Which skin condition occurs as an allergic reaction to mite eggs? a. Scabies c. Folliculitis b. Impetigo d. Pediculosis

a. In scabies mites penetrate the skin and deposits eggs. An allergic reaction can result from the presence of eggs, feces, and mite parts. Impetigo involves vesiculopustular lesions that develop a thick, honey-colored crust surrounded by erythema. Folliculitis is a small pustule at the hair follicle opening with minimal erythema. Pediculosis is lice.

To prevent lichenification related to chronic skin problems, what does the nurse encourage the patient to do? a. Use measures to control itching. b. Wear sterile gloves when touching the lesions. c. Use careful hand washing and safe disposal of soiled dressings. d. Use topical antibiotics with wet-to-dry dressings over the lesions.

a. Lichenification is thickening of the skin caused by chronic scratching or rubbing and can be prevented by controlling itching. It is not an infection, nor is it contagious, as the other options indicate.

A nurse caring for a disheveled patient with poor hygiene observes that the patient has small red lesions flush with the skin on the head and body. The patient complains of severe itching at the sites. For what should the nurse further assess the patient? a. Nits on the shafts of his head hair b. A history of sexually transmitted diseases c. The presence of ticks attached to the scalp d. The presence of burrows in the interdigital webs

a. Pediculosis (head lice and body lice) causes very small, red, noninflammatory lesions that progress to papular wheal-like lesions and cause severe itching. Lice live on the body as nits (tiny white eggs) that are firmly attached to hair shafts on the head and body. Burrows, especially in interdigital webs, are found with scabies.

What is a skin graft that is used to transfer skin and subcutaneous tissue to large areas of deep tissue destruction called? a. Skin flap c. Soft tissue extension b. Free graft d. Free graft with vascular anastomoses

a. Skin flaps as grafts include moving skin and subcutaneous tissue to another part of the body and are used to cover wounds with poor vascular beds, add padding, and cover wounds over cartilage and bone. Both types of free grafts include just skin and soft tissue extension involves placement of an expander under the skin, which stretches the skin over time to provide extra skin to cover the desired area.

A patient is a 78-year-old woman who has had chronic respiratory disease for 30 years. She weighs 212 lb (96.4 kg) and is 5 ft, 1 in (152.5 cm) tall. She has recently completed corticosteroid and antibiotic treatment for an exacerbation of her respiratory disease. Identify four specific predisposing factors for bacterial skin infection in this patient. a. b. c. d.

a. chronic disease; b. obesity; c. recent antibiotic therapy; d. recent corticosteroid therapy

Which statements characterize malignant melanomas (select all that apply)? a. Lesion is keratotic and firm b. Neoplastic growth of melanocytes c. Skin cancer with highest mortality rate d. Irregular color and asymmetric shape e. Frequently occurs on previously damaged skin

b, c, d. Actinic and firm lesions are actinic keratosis and squamous cell carcinoma. Squamous cell carcinoma frequently occurs in previously damaged skin.

The nurse plans care for a patient with a newly diagnosed malignant melanoma based on the knowledge that initial treatment may involve (select all that apply) a. shave biopsy. b. Mohs' surgery. c. surgical excision. e. localized radiation. f. fluorouracil (5-FU). g. topical nitrogen mustard.

b, c. In the early stages, surgical excision with a margin of 18. normal skin is the initial treatment for malignant melanoma. Mohs' surgery can also be used to treat malignant melanoma. Radiation may be used after excision for malignant melanoma, depending on staging of the disease. Topical nitrogen mustard may be used for treatment of cutaneous T-cell lymphoma.

What is the most common skin cancer and has pearly borders? a. Actinic keratosis b. Basal cell carcinoma c. Malignant melanoma d. Squamous cell carcinoma

b. Actinic keratosis is an irregularly shaped, flat, slightly erythematous papule with indistinct borders and an overlying hard keratotic scale or horn. Malignant melanoma tumors arise in melanocytes. Malignant melanoma is the deadliest skin cancer and has an increased risk in people with dysplastic nevus syndrome. Squamous cell carcinoma is a malignant neoplasm of keratinizing epidermal cells.

A female patient with chronic skin lesions of the face and arms tells the nurse that she cannot stand to look at herself in the mirror anymore because of her appearance. Based on this information, the nurse identifies which nursing diagnosis? a. Anxiety related to personal appearance b. Disturbed body image related to perception of unsightly lesions c. Social isolation related to decreased activities as a result of poor self-image d. Ineffective self-health management related to lack of knowledge of cover-up techniques

b. Defining characteristics for body image problems include verbalization of self-disgust and reluctance to look at lesions, as evidenced in this patient. Social isolation is indicated only if there is evidence of decreased social activities and of anxiety by verbalization of anxiety or frustration. Ineffective self-health management is indicated by evidence of a lack of self-care or understanding of the disease process.

Which description characterizes seborrheic keratosis? a. White patchy yeast infection b. Warty, irregular papules or plaques c. Excessive turnover of epithelial cells d. Deep inflammation of subcutaneous tissue

b. Seborrheic keratoses are irregularly round or oval shaped and are often verrucous papules or plaques. Candidiasis is a white patchy yeast infection. Cellulitis is a deep inflammation of subcutaneous tissue. Psoriasis is an excessive turnover of epithelial cells.

What is an appropriate intervention to promote debridement and removal of scales and crusts of skin lesions? a. Warm oatmeal baths b. Warm saline dressings c. Cool sodium bicarbonate baths d. Cool magnesium sulfate dressings

b. Tepid or warm solutions should be used when the purpose is debridement and saline is a common debridement solution. Baths are appropriate for debridement but sodium bicarbonate and oatmeal are used for pruritus.

Priority Decision: A patient is receiving chemotherapy. She calls the physician's office and says she is experiencing itching in her groin and under her breasts. What is the first nursing assessment that would be done before the nurse makes an appointment for the patient with the physician to determine the treatment? a. Her height and weight c. If chemotherapy was completed b. What the areas look like d. Culture and sensitivity of the areas

b. The appearance of candidiasis on the skin shows diffuse papular erythematous rash with pinpoint satellites around the affected area. Height and weight could show if the patient is obese but it would be better to ask if the areas affected are moist. The chemotherapy could contribute to candidiasis but it does not matter if the chemotherapy treatments are finished. Culture and sensitivity of the area may be ordered by the physician at the patient's appointment.

Priority Decision: A 46-year-old African American patient is scheduled to have a basal cell carcinoma on his cheek excised in the health care provider's office. What factor is most important for the nurse to obtain in the patient's history? a. Protected sun exposure b. Radiation treatment for acne c. Prior treatments for the lesion d. Exposure to harsh irritants such as ammonia

b. Thirty years ago, when the patient was a teenager, radiation therapy was used to treat cystic acne with the result that many of these patients now have developed basal cell carcinoma. For a person with dark skin, radiation therapy is a higher risk factor for skin cancer than exposure to the sun or other irritants.

What should the nurse include in the instructions for a patient with urticaria? a. Apply topical benzene hexachloride. b. Avoid contact with the causative agent. c. Gradually expose the area to increasing amounts of sunlight. d. Use over-the-counter antihistamines routinely to prevent the condition.

b. Urticaria is inflammation and edema in the upper dermis, most commonly caused by histamine released during an antibody-allergen reaction. The best treatment for all types of allergic dermatitis is avoidance of the allergen. Sunlight and warmth would increase the edema and inflammation. Antihistamines may be used for an acute outbreak but not to prevent the dermatitis. Topical benzene hexachloride is used to treat pediculosis.

A nurse admits that a client with newly diagnosed DM. When reviewing the client's lab work, the nurse notes that the result consistent with diabetic ketoacidosis is what?

bicarbonate level of 12.

Which skin conditions are more common in immunosuppressed patients (select all that apply)? a. Acne b. Lentigo c. Candidiasis d. Herpes zoster e. Herpes simplex 1 f. Kaposi sarcoma

c, e, f. Patients who are immunocompromised are at an increased risk for candidiasis (a fungal infection), herpes simplex 1 (caused by a virus), and Kaposi sarcoma (vascular 22. lesions on the skin, mucous membranes, and viscera with wide range of presentation). The other options are not at increased risk with immunosuppression. Acne is caused by inflammation of sebaceous glands. Lentigo (also called "liver spots" or "age spots") is caused by an increased number of normal melanocytes in the basal layer of epidermis. Herpes zoster, which is caused by an activation of the varicella- zoster virus, is a group of vesicles and pustules resembling chickenpox located in a linear distribution along a dermatome.

A patient with a contact dermatitis is treated with calamine lotion. What is the rationale for using this base for a topical preparation? a. A suspension of oil and water to lubricate and prevent drying b. An emulsion of oil and water used for lubrication and protection c. Insoluble powders suspended in water that leave a residual powder on the skin d. A mixture of a powder and ointment that causes drying when moisture is absorbed

c. A lotion is a suspension of insoluble powders in water, which has cooling and drying properties, useful when itching is present. Creams and ointments have an oil and water base that lubricates and protects skin whereas a paste is a mixture of powder and ointment.

Which skin condition would be treated with laser surgery? a. Preauricular lesion c. Obesity with subcutaneous fat accumulation b. Redundant soft tissue conditions d. Fine wrinkle reduction or facial lesion removal

d. A facelift is used for preauricular lesions and redundant soft tissue reduction. Liposuction is used for obesity with subcutaneous fat accumulation.

What is the name for papillomavirus infection seen on the skin? a. Furuncle b. Carbuncle c. Erysipelas d. Plantar wart

d. A plantar wart is caused by human papillomavirus (HPV). A furuncle is a deep skin infection with staphylococci around the hair follicle. A carbuncle is multiple, interconnecting furuncles. Erysipelas is superficial cellulitis primarily involving the dermis.

What characteristic is commonly seen with dysplastic nevus syndrome? a. Associated with sun exposure b. Precursor of squamous cell carcinoma c. Slow-growing tumor with rare metastasis d. Lesion has irregular color and asymmetric shape

d. Dysplastic nevus syndrome involves atypical moles with irregular borders and various shades of color.

A patient with psoriasis is being treated with psoralen plus UVA light (PUVA) phototherapy. During the course of therapy, for what duration should the nurse teach the patient to wear protective eyewear that blocks all UV rays? a. Continuously for 6 hours after taking the medication b. Until the pupils are able to constrict on exposure to light c. For 12 hours following treatment to prevent retinal damage d. For 24 hours following treatment when outdoors or when indoors near a bright window

d. Psoralen is absorbed by the lens of the eye and eyewear that blocks 100% of UV light must be used for 24 hours after taking the medication. Because UVA penetrates glass, the eyewear must also be worn indoors when near a bright window. Psoralen does not affect the accommodative ability of the eye.

A nurse is assessing a client with Grave's disease. The nurse should expect the client to report what?

difficulty sleeping

The nurse is teaching a client with DM I about early manifestations of hypoglycemia. What should the nurse be sure to include in her teaching?

drowsiness


Conjuntos de estudio relacionados

PSY 101 Macmillan Learning Launchpad

View Set

Clinical Psych Final Exam Review

View Set

MAR 5625 - Marketing Research and Analytics - Chapter 7

View Set

final consumers in global market

View Set